Chapter 45: Caring for Clients with Disorders of the Upper Gastrointestinal Tract, GI Pharm Practice Questions, Med Surg Chapter 23: Nursing Management: Patients With Gastric and Duodenal Disorders: PREPU, Chapter 46: Mgt With Gastric and Duodenal Di...

¡Supera tus tareas y exámenes ahora con Quizwiz!

Morbid obesity is defined as being how many pounds over the person's ideal body weight?

100 Morbid obesity is a term applied to people who are more than two times their ideal body weight or whose body weight index (BMI) exceeds 30 kg/m2. Another definition of morbid obesity is body weight that is more than 100 pounds greater than the ideal body weight.

Rebound hypoglycemia is a complication of parenteral nutrition caused by

feedings stopped too abruptly.

A client is in the initial stages of oral cancer diagnosis and is frightened about the side effects of treatment and subsequent prognosis. The client has many questions regarding this type of cancer and asks where oral cancer typically occurs. What is the nurse's response?

floor of the mouth

Using the the client's weight in pounds (215) is divided by the height in inches squared (68 inches squared) and then multiplied by 703.

formula for BMI,

A client with morbid obesity is being scheduled for malabsorptive bariatric surgery. The nurse would provide teaching about which procedure? A. Roux-en-Y gastric bypass B. sleeve gastrectomy C. biliopancreatic diversion D. gastric banding

gastric banding

Most clients are asymptomatic during the early stage of the disease. Men have a higher incidence of gastric cancer. The prognosis is poor because the diagnosis is usually made late because most clients are asymptomatic during the early stage. Most cases of gastric cancer are discovered only after local invasion has advanced or metastases are present.

gastric cancer

hyposecretion of stomach acid.

gastric ulcer evidences

The nurse is conducting a community education program on peptic ulcer disease prevention. The nurse includes that the most common cause of peptic ulcers is: stress and anxiety. gram-negative bacteria. alcohol and tobacco. ibuprofen and aspirin.

gram-negative bacteria. The nurse should include that the most common cause of peptic ulcers is gram-negative bacteria (Helicobacter pylori).

Which of the following is an inaccurate clinical manifestation associated with hemorrhage?

Bradycardia

A patient comes to the clinic with the complaint, "I think I have an ulcer." What is a characteristic associated with peptic ulcer pain that the nurse should inquire about? (Select all that apply.)

Burning sensation localized in the back or mid-epigastrium Feeling of emptiness that precedes meals from 1 to 3 hours Severe gnawing pain that increases in severity as the day progresses

the term for wasting syndrome.A general state of ill health involving marked weight loss and muscle loss

Chachexia

include serologic testing for antibodies against the H. pylori antigen, stool antigen test, and urea breath test.

Less invasive diagnostic measures for detecting H. pylori

decreases gastric acid by slowing the hydrogen-potassium adenosine triphosphatase pump on the surface of the parietal cells.

Omeprazole

A nurse practitioner prescribes drug therapy for a patient with peptic ulcer disease. Choose the drug that can be used for 4 weeks and has a 90% chance of healing the ulcer. Ranitidine (Zantac) Cimetidine (Tagamet) Famotidine (Pepcid) Omeprazole (Prilosec)

Omeprazole (Prilosec) Omeprazole (Prilosec) is a proton pump inhibitor that, if used according to the health care provider's directions, will result in healing in 90% of patients. The other drugs are H2 receptor antagonists that need to be used for 6 weeks.

A nurse practitioner prescribes drug therapy for a patient with peptic ulcer disease. Choose the drug that can be used for 4 weeks and has a 90% chance of healing the ulcer. A. Omeprazole (Prilosec) B. Cimetidine (Tagamet) C. Famotidine (Pepcid) D. Ranitidine (Zantac)

Omeprazole (Prilosec) Omeprazole (Prilosec) is a proton pump inhibitor that, if used according to the health care provider's directions, will result in healing in 90% of patients. The other drugs are H2 receptor antagonists that need to be used for 6 weeks.

A patient asks the home health nurse from what the distressing symptoms of dumping syndrome result. What physiological occurrence should the nurse explain?

Osmotic transport of extracellular fluid into the gastrointestinal tract

The nurse is assessing a client with a bleeding gastric ulcer. When examining the client's stool, which of the following characteristics would the nurse be most likely to find?

Black and tarry appearance

Which medication is classified as a histamine-2 receptor antagonist

Famotidine

Which medication is classified as a histamine-2 receptor antagonist?

Famotidine

is a histamine-2 receptor antagonist.

Famotidine (Pepcid)

The nurse is caring for a comatose patient and administering gastrostomy feedings. What does the nurse understand is the reason that gastrostomy feedings are preferred to nasogastric (NG) feedings in the comatose patient?

Gastroesophageal sphincter is intact, lessening the possibility of regurgitation.

A patient is experiencing painful, inflamed, and swollen gums, and when brushing the teeth, the gums bleed. What common disease of the oral tissue does the nurse understand these symptoms indicate?

Gingivitis

The nurse advises the patient who has just been diagnosed with acute gastritis to:

Refrain from food until the GI symptoms subside.

A client with a peptic ulcer is about to begin a therapeutic regimen that includes a bland diet, antacids, and famotidine (Pepcid). Before the client is discharged, the nurse should provide which instruction?

"Avoid aspirin and products that contain aspirin." The nurse should instruct the client to avoid aspirin because it's a gastric irritant and should not be taken by clients with peptic ulcer to prevent further erosion of the stomach lining. The client should eat small, frequent meals rather than three large ones. Antacids and ranitidine prevent acid accumulation in the stomach; they should be taken even after symptoms subside. Caffeine should be avoided because it increases acid production in the stomach.

To prevent gastroesophageal reflux in a client with hiatal hernia, the nurse should provide which discharge instruction

"Avoid coffee and alcoholic beverages."

A patient being treated for pyloric obstruction has a nasogastric (NG) tube in place to decompress the stomach. The nurse routinely checks for a residual amount that would indicate obstruction. Choose that amount.

450 mL

A client weighs 215 lbs and is 5' 8" tall. The nurse would calculate this client's body mass index (BMI) as which of the following? 19.5 24.8 32.7 44.9

32.7 Using the formula for BMI, the client's weight in pounds (215) is divided by the height in inches squared (68 inches squared) and then multiplied by 703. The result would be 32.7.

The nurse is caring for a patient who has been diagnosed with gastritis. To promote fluid balance when treating gastritis, the nurse knows that what minimal daily intake of fluids is required?

1.5 L

A client reports to the clinic, stating that she rapidly developed headache, abdominal pain, nausea, hiccuping, and fatigue about 2 hours ago. For dinner, she ate buffalo chicken wings and beer. Which of the following medical conditions is most consistent with the client's presenting problems?

Acute gastritis

The nurse practitioner suspects that a patient may have a gastric ulcer after completing a history and physical exam. Select an indicator that can be used to help establish the distinction. Amount of hydrochloric acid (HCL) secretion in the stomach Sensitivity to the use of nonsteroidal anti-inflammatory drugs (NSAIDs) Presence of H. pylori Patient's age

Amount of hydrochloric acid (HCL) secretion in the stomach

An older adult patient with a diagnosis of chronic gastritis has achieved acceptable control of his condition with the use of an H2 receptor antagonist. This patient's symptom control is a result of what therapeutic action of this drug? A.Activation of the gastric buffer system and release of alkaline gastric secretions B. The occlusion of parietal cells C. An increase in the pH of gastric secretions D. A decrease in HCl production by parietal cells

An increase in the pH of gastric secretions D. A decrease in HCl production by parietal cells

is associated with duodenal ulcers.

Chronic renal failure

A client has given a confirmed diagnosis of gastric cancer. Two more procedures may be performed to assess tumor depth and lymph node involvement and surgical resectability. Which two are the procedures? Choose the two that apply.

Computed tomography (CT) Endoscopic ultrasound Esophagogastroduodenoscopy for biopsy and cytologic washings is the diagnostic study of choice, and a barium x-ray examination of the upper GI tract may also be performed. Endoscopic ultrasound is an important tool to assess tumor depth and any lymph node involvement. Pelvic ultrasound is not used to confirm the diagnosis of gastric cancer.

by pain and a pulling or popping feeling at the surgical site.

Dehiscence of the surgical wound is characterized

Clients with Type O blood are at higher risk for which of the following GI disorders? Gastric cancer Duodenal ulcers Esophageal varices Diverticulitis

Duodenal ulcers Familial tendency also may be a significant predisposing factor. People with blood type O are more susceptible to peptic ulcers than are those with blood type A, B, or AB. Blood type is not a predisposing factor for gastric cancer, esophageal varices, and diverticulitis.

The nursing student approaches his instructor to discuss the plan of care for his client diagnosed with peptic ulcer disease. The student asks what is the most common site for peptic ulcer formation? The instructor would state which one of the following? Duodenum Esophagus Pylorus Stomach

Duodenum

The nursing student approaches his instructor to discuss the plan of care for his client diagnosed with peptic ulcer disease. The student asks what is the most common site for peptic ulcer formation? The instructor would state which one of the following? A. Duodenum B.Stomach C. Esophagus D. Pylorus

Duodenum

Which term refers to the first portion of the small intestine?

Duodenum

A physician suspects that a client has peptic ulcer disease. With which of the following diagnostic procedures would the nurse most likely prepare to assist?

Endoscopy Barium study of the upper GI tract may show an ulcer; however, endoscopy is the preferred diagnostic procedure because it allows direct visualization of inflammatory changes, ulcers, and lesions. Through endoscopy, a biopsy of the gastric mucosa and of any suspicious lesions can be obtained. Endoscopy may reveal lesions that, because of their size or location, are not evident on x-ray studies. Less invasive diagnostic measures for detecting H. pylori include serologic testing for antibodies against the H. pylori antigen, stool antigen test, and urea breath test.

The nurse is caring for a patient who has been diagnosed with a peptic ulcer. The patient asks the nurse what a peptic ulcer is. Which of the following best describes a peptic ulcer? A) Inflammation of the lining of the stomach B) Erosion of the lining of the stomach or intestine C) Bleeding from the mucosa in the stomach D) Viral invasion of the stomach wall

Erosion of the lining of the stomach or intestine A peptic ulcer is erosion of the lining of the stomach or intestine. It is not inflammation, bleeding, or a viral invasion of the stomach wall.

Which of the following is the most common complication associated with peptic ulcer? Hemorrhage Vomiting Elevated temperature Abdominal pain

Hemorrhage Hemorrhage, the most common complication, occurs in 28% to 59% of patients with peptic ulcers. Vomiting, elevated temperature, and abdominal pain are not the most common complications of a peptic ulcer.

Which of the following dietary guidelines should be followed following bariatric surgery? Select all that apply.

Include two protein snacks per day. Eat slowly. Eat three meals per day.

The client has been taking famotidine (Pepcid) at home. The nurse prepares a teaching plan for the client indicating that the medication acts primarily to achieve which of the following? Inhibit gastric acid secretions. Neutralize acid in the stomach. Shorten the time required for digestion in the stomach. Improve the mixing of foods and gastric secretions.

Inhibit gastric acid secretions.

The nurse is aware that hemorrhage is a common complication of peptic ulcer disease. Therefore, assessment for indicators of bleeding is an important nursing responsibility. Which of the following are indicators of bleeding? Select all that apply.

Melena Tachypnea Thirst Mental confusion

Amanda Ford, a 53-year-old actress, is being seen by a physician in your oncology group due to her oral cancer. In the initial stages of diagnosis, she is frightened about her treatment and subsequent prognosis. During your client education session following the physician's examination, what do you indicate as the early signs of oral cancer?

No symptoms

A nurse is caring for a client with active upper GI bleeding. What is the appropriate diet for this client during the first 24 hours after admission?

Nothing by mouth

Peptic ulcer disease occurs more frequently in people with which blood type? A B AB O

O People with blood type O are more susceptible to peptic ulcers than those with blood type A, B, or AB.

Which medication classification represents a proton (gastric acid) pump inhibitor? Omeprazole Sucralfate Famotidine Metronidazole

Omeprazole (Prilosec) Omeprazole decreases gastric acid by slowing the hydrogen-potassium adenosine triphosphatase pump on the surface of the parietal cells. Sucralfate is a cytoprotective drug. Famotidine is a histamine-2 receptor antagonist. Metronidazole is an antibiotic, specifically an amebicide.

Omeprazole (Prilosec) is a proton pump inhibitor that, if used according to the health care provider's directions, will result in healing in 90% of patients. The other drugs are Ranitidine (Zantac) Cimetidine (Tagamet) Famotidine (Pepcid): H2 receptor antagonists that need to be used for 6 weeks.

Peptic Ulcer Disease Treatment

A patient has just been diagnosed with acute gastritis. What would be the nursing care needed by this patient? A) Teaching regarding nutrition B) Advising the patient not to smoke C) Recommending bed rest D) Physical and emotional support

Physical and emotional support For acute gastritis, the nurse provides physical and emotional support and helps the patient manage the symptoms, which may include nausea, vomiting, heartburn, and fatigue. The scenario describes a newly diagnosed patient. Nursing care of this patient would not include teaching regarding nutrition or smoking at this time. Also, the nurse would not recommend bed rest.

The nurse is caring for a patient who is suspected to have developed a peptic ulcer hemorrhage. Which action would the nurse perform first? Place the patient in a recumbent position with the legs elevated. Prepare a peripheral and central line for intravenous infusion. Assess vital signs. Call the physician.

Place the patient in a recumbent position with the legs elevated.

The client may have nausea and vomiting, constipation, epigastric fullness, anorexia, and, later, weight loss.

Pyloric obstruction, also called gastric outlet obstruction (GOO), signs

A patient with a peptic ulcer has asked the nurse for help in selecting their dinner menu. Which beverage would be most appropriate for this patient? A) Vanilla milkshake B) Decaffeinated coffee C) Unsweetened iced tea D) Room-temperature grape juice

Room-temperature grape juice Coffee and tea, whether caffeinated or decaffeinated, stimulate gastric acid secretion. The same is true of foods rich in milk and cream. Over-secretion of acid and hypermotility of the GI tract can be decreased by avoiding extremes of temperature in food and beverages.

The nurse is completing a health history on a patient whose diagnosis is chronic gastritis. Which of the data below should the nurse consider most significantly related to the etiology of the patient's health problem? A) Consumes one or more protein drinks daily. B) Takes over-the-counter antacids frequently throughout the day. C) Smokes two packs of cigarettes daily. D) Reports a history of social drinking on a weekly basis.

Smokes two packs of cigarettes daily. Nicotine reduces secretion of pancreatic bicarbonate, which inhibits neutralization of gastric acid. Antacid use is a response to experiencing symptoms of gastritis, not the etiology of gastritis. Alcohol ingestion can lead to gastritis; however, this generally occurs in patients with a history of daily consumption of alcohol on a daily basis. Protein drinks do not result in gastric inflammation

A patient taking metronidazole (Flagyl) for the treatment of H. pylori states that the medication is causing nausea. What suggestion can the nurse provide to the patient to alleviate this problem?

Take the medication with meals to decrease the nausea. Metronidazole (Flagyl) should be administered with meals to decrease GI upset.

may cause a photosensitivity reaction in clients. The nurse should caution the client to use sunscreen when taking this drug. Dairy products can reduce the effectiveness of tetracycline, so the nurse should not advise him or her to take the medication with milk.

Tetracycline

A client with a peptic ulcer is diagnosed with Heliobacter pylori infection. The nurse is teaching the client about the medications prescribed, including metronidazole (Flagyl), omeprazole (Prilosec), and clarithromycin (Biaxin). Which statement by the client indicates the best understanding of the medication regimen?

The medications will kill the bacteria and stop the acid production."

is gram-negative bacteria (Helicobacter pylori).

The nurse should include that the most common cause of peptic ulcers

Which statement correctly identifies a difference between duodenal and gastric ulcers?

Vomiting is uncommon in clients with duodenal ulcers.

What is the pH of gastric aspirate

acidic

A nurse is teaching a group of middle-aged men about peptic ulcers. When discussing risk factors for peptic ulcers, the nurse should mention: a sedentary lifestyle and smoking. a history of hemorrhoids and smoking. alcohol abuse and a history of acute renal failure. alcohol abuse and smoking.

alcohol abuse and smoking. The nurse should mention that risk factors for peptic (gastric and duodenal) ulcers include alcohol abuse, smoking, and stress. A sedentary lifestyle and a history of hemorrhoids aren't risk factors for peptic ulcers. Chronic renal failure, not acute renal failure, is associated with duodenal ulcers.

The nurse is conducting a community education program on peptic ulcer disease prevention. The nurse includes that the most common cause of peptic ulcers is: A. ibuprofen and aspirin. B. alcohol and tobacco. C. gram-negative bacteria. D.stress and anxiety.

gram-negative bacteria. The nurse should include that the most common cause of peptic ulcers is gram-negative bacteria (Helicobacter pylori).

ingestion of strong acid or alkali, which may cause the mucosa to become gangrenous or to perforate.

severe form of acute gastritis is caused by

The nurse interprets hematemesis. Other signs include tachycardia, hypotension, and oliguria/anuria.

sign/symptom of possible hemorrhage from an ulcer

The most significant complication related to continuous tube feedings is

the potential for aspiration.

The nurse is inserting a Levin tube for a patient for gastric decompression. The tube should be inserted to 6 to 10 cm beyond what length?

The distance measured from the tip of the nose to the earlobe and from the earlobe to the xiphoid process

Which is an accurate statement regarding gastric cancer?

The incidence of stomach cancer continues to decrease in the United States. While the incidence in the United States continues to decrease, gastric cancer still accounts for 10,700 deaths annually. While gastric cancer deaths occasionally occur in younger people, most occur in people older than 40 years of age. Males have a higher incidence of gastric cancers than females. More accurately, a diet high in smoked foods and low in fruits and vegetables may increase the risk of gastric cancer.

A client with a peptic ulcer is diagnosed with Heliobacter pylori infection. The nurse is teaching the client about the medications prescribed, including metronidazole (Flagyl), omeprazole (Prilosec), and clarithromycin (Biaxin). Which statement by the client indicates the best understanding of the medication regimen

The medications will kill the bacteria and stop the acid production."

A nurse is preparing to discharge a patient after gastric surgery. What is an appropriate discharge outcome for this patient? A) The patient's bowel movements are loose. B) The patient eats three meals a day. C) The patient maintains or gains weight. D) The patient consumes a diet high in calcium.

The patient maintains or gains weight. Expected outcomes for the patient following gastric surgery include daily weights to ensure that the patient is maintaining or gaining weight, experiencing no excessive diarrhea, and tolerating six small meals a day. Patients may require vitamin B12 supplementation by the intramuscular route and do not require a diet excessively rich in calcium.

A patient with gastroesophageal reflux disease (GERD) has a diagnosis of Barrett's esophagus and has been admitted to a medical unit. The nurse is writing a care plan for this patient. What information is essential to include?

He will need to undergo an upper endoscopy every 6 months to detect malignant changes.

The nurse is assessing a client with an ulcer for signs and symptoms of hemorrhage. The nurse interprets which condition as a sign/symptom of possible hemorrhage?

Hematemesis

the most common complication, occurs in 10% to 20% of clients with peptic ulcers. Bleeding may be manifested by hematemesis or melena..

Hemorrhage

A critical care nurse is closely monitoring a patient who has recently undergone surgical repair of a bleeding peptic ulcer. The nurse should prioritize assessments of which of the following signs and symptoms of a recurrence of hemorrhage? Restlessness and cyanosis Hypotension and tachycardia Bradypnea and pursed lip breathing Peripheral and pulmonary edema

Hypotension and tachycardia Rebleeding has multiple manifestations. However, an increase in heart rate and decrease in blood pressure are key signs of a hemorrhage that are present in nearly all patients who are experiencing rebleeding.

During assessment of a patient with gastritis, the nurse practitioner attempts to distinguish acute from chronic pathology. One criteria, characteristic of gastritis would be the:

Immediacy of the occurrence.

The nurse recognizes that the client diagnosed with a duodenal ulcer will likely experience

pain 2 to 3 hours after a meal.

reduced if the client avoids smoking, coffee (including decaffeinated coffee) and other caffeinated beverages, and alcohol. It is important to counsel the client to eat meals at regular times and in a relaxed setting and to avoid overeating.

peptic ulcer disease nonpharmacologic ways to prevent recurrence

is sometimes associated with autoimmune disease, such as pernicious anemia, but not as a cause of the anemia.

Chronic gastritis

Helicobacter pylori is implicated in the development of peptic ulcers.

Chronic gastritis caused by

Which is an accurate statement regarding cancer of the esophagus?

Chronic irritation of the esophagus is a known risk factor

Which of the following clients is at highest risk for peptic ulcer disease?

Client with blood type O Clients with blood type O are more susceptible to peptic ulcers than those with blood types A, B, and AB.

Peptic ulcer disease occurs more frequently in people with which blood type? Client with blood type A Client with blood type B Client with blood type AB Client with blood type O

Client with blood type O People with blood type O are more susceptible to peptic ulcers than those with blood type A, B, or AB.

A nurse is performing discharge teaching with a client who had a total gastrectomy. Which statement indicates the need for further teaching?

"I will have to take vitamin B12 shots up to 1 year after surgery

Which of the following manifestations are associated with a deficiency of vitamin B12? Select all that apply.

-Pernicious anemia -Macrocytic anemia -Thrombocytopenia Decreased vitamin B12 can result in pernicious anemia, macrocytic anemia, and thrombocytopenia. Decreased iron can result in lethargy and loss of hair.

The health care provider prescribes a combination of three drugs to treat peptic ulcer disease. The nurse, preparing to review the drug actions and side effects with the patient, understands that the triple combination should be in which order? Antibiotics, prostaglandin E1 analogs, and bismuth salts Proton pump inhibitors, prostaglandin E1 analogs, and bismuth salts Bismuth salts, antibiotics, and proton pump inhibitors Prostaglandin E1 analogs, antibiotics, and proton pump inhibitors

DELETE

Symptoms associated with pyloric obstruction include all of the following except: Anorexia Diarrhea Nausea and vomiting Epigastric fullness

Diarrhea

A patient who had a Roux-en-Y bypass procedure for morbid obesity ate a chocolate chip cookie after a meal. After ingestion of the cookie, the patient complained of cramping pains, dizziness, and palpitation. After having a bowel movement, the symptoms resolved. What should the patient be educated about regarding this event?

Dumping syndrome Dumping syndrome is an unpleasant set of vasomotor and GI symptoms that occur in up to 76% of patients who have had bariatric surgery. Early symptoms include a sensation of fullness, weakness, faintness, dizziness, palpitations, diaphoresis, cramping pains, and diarrhea. These symptoms resolve once the intestine has been evacuated (i.e., with defecation).

A client has a new order for metoclorpramide (Reglan). The nurse identifies that this medication should not be used long term and only in cases where all other options have been exhausted. This is because this medication has what type of potential side effect?

Extrapyramidal

is useful for treating and preventing ulcers and managing gastroesophageal reflux disease. It functions by inhibiting the action of histamine at the H-2 receptor site located in the gastric parietal cells, thus inhibiting gastric acid secretion.

Famotidine (Pepcid)

Rebleeding may occur from a peptic ulcer and often warrants surgical interventions. Signs of bleeding include which of the following? Mental confusion Bradycardia Bradypnea Hypertension

Mental confusion

Rebleeding may occur from a peptic ulcer and often warrants surgical interventions. Signs of bleeding include which of the following?

Mental confusion Signs of bleeding include tachycardia, tachypnea, hypotension, mental confusion, thirst, and oliguria.

The nurse in the ED admits a client with suspected gastric outlet obstruction. The client's symptoms include nausea and vomiting. The nurse anticipates that the physician will issue which order? Pelvic x-ray Stool specimen Nasogastric tube insertion Oral contrast

Nasogastric tube (NG tube)

decompress the stomach.

Nasogastric tube (NG tube)

The nurse in the ED admits a client with suspected gastric outlet obstruction. The client's symptoms include nausea and vomiting. The nurse anticipates that the physician will issue which order

Nasogastric tube insertion

The nurse in the ED admits a client with suspected gastric outlet obstruction. The client's symptoms include nausea and vomiting. The nurse anticipates that the physician will issue which order? A. Oral contrast B. Stool specimen C. Pelvic x-ray D. Nasogastric tube insertion

Nasogastric tube insertion

The nurse assists the client to identify stressful or exhausting situations. A hectic lifestyle and an irregular schedule may aggravate symptoms and interfere with regular meals taken in relaxed settings along with the regular administration of medications. The client may benefit from regular rest periods during the day, at least during the acute phase of the disease. Biofeedback, hypnosis, behavior modification, massage, or acupuncture may be helpful.

Peptic Ulcer Disease

presents with a rigid, boardlike abdomen, tenderness, and fever.

Peritonitis

include tachycardia, tachypnea, hypotension, mental confusion, thirst, and oliguria.

Signs of bleeding

cool skin, confusion, increased heart rate, labored breathing, and blood in the stool.

Signs of hemorrhage following surgery include

Which are accurate clinical manifestations associated with hemorrhage? Select all that apply.

Tachycardia

complete rest for the GI tract, placing the client in a recumbent position with the legs elevated, blood transfusions, and gastric lavage with saline solution.

Treatment of hemorrhage includes

A team approach is the best approach to discharge planning for a patient who has had gastric surgery. What is helpful to the patient and family to reinforce discharge teaching? A) Home health nurse re-teaching material in patient's home B) Scheduled visits from speech therapist C) Scheduled phone calls from social worker D) Video instructions about meals and activities

Video instructions about meals and activities Written or video instructions about meals, activities, medications, and follow-up care are helpful. The home health nurse may or may not be involved in this patient's care if the patient is not on tube feedings. A speech therapist is not necessary in gastric surgery. The social worker would schedule visits, if necessary, not phone calls.

Which of the following is the most common type of diverticulum?

Zenker's diverticulum

consists of severe peptic ulcers, extreme gastric hyperacidity, and gastrin-secreting benign or malignant tumors of the pancreas. Diarrhea and steatorrhea may be evident. The client may have co-existing parathyroid adenomas or hyperplasia and may therefore exhibit signs of hypercalcemia.

Zollinger-Ellison syndrome (ZES)

A nurse is caring for a client who underwent a subtotal gastrectomy. To manage dumping syndrome, the nurse should advise the client to:

drink liquids only between meals.

Signs and symptoms include vertigo, tachycardia, syncope, sweating, pallor, palpitations, diarrhea, nausea, and the desire to lie down.

dumping syndrome

A hypersecretion of stomach acid

duodenal ulcer is characterized by

The nurse is aware that hemorrhage is a common complication of peptic ulcer disease. Therefore, assessment for indicators of bleeding is an important nursing responsibility. Which of the following are indicators of bleeding? Select all that apply.

-Melena -Tachypnea -Thirst -Mental confusion Tachycardia and oliguria would be present with bleeding as the body tries to compensate for blood loss.

After teaching a client who has had a Roux-en-Y gastric bypass, which client statement indicates the need for additional teaching?

"I need to drink 8 ounces of water before eating." After a Roux-en-Y gastric bypass, the client should not drink fluids with meals, withholding fluids for 15 minutes before eating to 90 minutes after eating. Chewing foods slowly and thoroughly, keeping total serving sizes to less than 1 cup, and choosing foods such as breads, cereals, and grains that provide less than 2 g of fiber per serving.

An obese male patient has sought advice from the nurse about the possible efficacy of medications in his efforts to lose weight. What should the nurse teach the patient about pharmacologic interventions for the treatment of obesity? 1. "Medications are usually reserved for people who have had unsuccessful bariatric surgery." 2. "Medications may be of some use, but they don't tend to resolve obesity on their own." 3. "Medications are an excellent option for individuals who prefer not to exercise or reduce their food intake." 4. "Medications have the potential to reduce hunger but they rarely result in weight loss."

"Medications may be of some use, but they don't tend to resolve obesity on their own." Medications for obesity rarely result in a loss of more than 10% of total body weight. They are not intended as a substitute for exercise or a healthy diet. They are not solely intended for those individuals who have undergone bariatric surgery.

A nurse is teaching a client who has experienced an episode of acute gastritis and knows further education is necessary when the client makes the following statement:

"My appetite should come back tomorrow." The gastric mucosa is capable of repairing itself after an episode of gastritis. As a rule, the client recovers in about 1 day, although the appetite may be diminished for an additional 2 or 3 days. Acute gastritis is also managed by instructing the client to refrain from alcohol and food until symptoms subside. When the client can take nourishment by mouth, a nonirritating diet is recommended.

A client with a peptic ulcer is diagnosed with Helicobacter pylori infection. The nurse is teaching the client about the medications prescribed, including metronidazole (Flagyl), omeprazole (Prilosec), and clarithromycin (Biaxin). Which statement by the client indicates the best understanding of the medication regimen?

"The medications will kill the bacteria and stop the acid production."

A client admitted for treatment of a gastric ulcer is being prepared for discharge. The client will follow a regimen of antacid therapy. Discharge teaching should include which instructions? Choose all that apply.

-"Continue to take antacids even if your symptoms subside." -"You may be prescribed H2-receptor antagonists for up to 1 year." The client is advised to adhere to and complete the medication regimen to ensure complete healing of the ulcer. Because most clients become symptom-free within 1 week, the nurse stresses the importance of following the prescribed regimen so that the healing process can continue uninterrupted and the return of chronic ulcer symptoms can be prevented. Maintenance dosages of H2-receptor antagonists are usually recommended for 1 year. Taking antacids concomitantly with other drugs should be avoided. For best results antacids should be taken 1 hour before or 2 hours after meals. Antacids will not make the client sleepy.

A client is prescribed a histamine (H2)-receptor antagonist. The nurse understands that the following are H2-receptor antagonists. Choose all that apply.

-Nizatidithene (Axid) -Ranitidine (Zantac) -Famotidine (Pepcid) -Cimetidine (Tagamet) H2-receptor antagonists suppress secretion of gastric acid, alleviate symptoms of heartburn, and assist in preventing complications of peptic ulcer disease. These medications also suppress gastric acid secretions and are used in active ulcer disease, erosive esophagitis, and pathological hypersecretory conditions. The other medications listed are proton-pump inhibitors.

You are writing a plan of care for a patient newly diagnosed with gastric cancer. What would the major goals for this patient include? (Mark all that apply.) A) Eat three nourishing meals daily B) Maintain adequate psychosocial support C) Relief of pain D) Adjustment to the diagnosis E) Reduced anxiety

-Relief of pain Adjustment to the diagnosis -Reduced anxiety The major goals for the patient may include reduced anxiety, optimal nutrition, relief of pain, and adjustment to the diagnosis and anticipated lifestyle changes. Major goals would not include eating three meals a day or maintaining adequate psychosocial support.

The nurse is caring for a client who has just returned from the PACU after surgery for peptic ulcer disease. For what potential complications does the nurse know to monitor? Select all that apply.

Hemorrhage Perforation Penetration Pyloric obstruction

A client with morbid obesity and a history of severe sleep apnea and severe diabetes is being considered for bariatric surgery. When reviewing the client's medical record, the nurse would identify that which body mass index (BMI) would meet the criteria for such surgery? A. 32 kg/m2 B. 30kg/m2 C. 34 kg/m2 D. 36 kg/m2

36 kg/m2

A client who is being treated for pyloric obstruction has a nasogastric (NG) tube in place to decompress the stomach. The nurse routinely checks for obstruction which would be indicated by what amount?

450 mL

A patient has come to the clinic complaining of pain just above her umbilicus. When assessing the patient, the nurse notes Sister Mary Joseph's nodules. The nurse knows that this is a sign of what? A) A GI malignancy B) Dumping syndrome C) Peptic ulcer disease D) Esophageal/gastric obstruction

A GI malignancy Palpable nodules around the umbilicus, called Sister Mary Joseph's nodules, are a sign of a GI malignancy, usually a gastric cancer. This would not be a sign of dumping syndrome, peptic ulcer disease, or esophageal/gastric obstruction.

A patient is scheduled for a Billroth I procedure for ulcer management. What does the nurse understand will occur when this procedure is performed?

A partial gastrectomy is performed with anastomosis of the stomach segment to the duodenum.

A patient is scheduled for a Billroth I procedure for ulcer management. What does the nurse understand will occur when this procedure is performed? A sectioned portion of the stomach is joined to the jejunum. The antral portion of the stomach is removed and a vagotomy is performed. A partial gastrectomy is performed with anastomosis of the stomach segment to the duodenum. The vagus nerve is cut and gastric drainage is established.

A partial gastrectomy is performed with anastomosis of the stomach segment to the duodenum. A Billroth I procedure involves removal of the lower portion of the antrum of the stomach (which contains the cells that secrete gastrin) as well as a small portion of the duodenum and pylorus. The remaining segment is anastomosed to the duodenum.

A patient comes to the clinic complaining of pain in the epigastric region. The nurse suspects that the patient's pain is related to a peptic ulcer when the patient states the pain is relieved by what? A) Eating B) Drinking milk C) Suppressing emesis D) Having a bowel movement

A)Eating Taking antacids, eating, or vomiting often relieves the pain. Pain occurs about 2 hours after eating. Milk is contraindicated in relieving peptic ulcer pain.

DRUG CLASS THAT CAN CAUSE LIVER FAILURE

ACETAMINOPHEN

A patient tells the nurse that it feels like food is "sticking" in the lower portion of the esophagus. What motility disorder does the nurse suspect these symptoms indicate?

Achalasia

A client comes to the clinic after developing a headache, abdominal pain, nausea, hiccuping, and fatigue about 2 hours ago. The client tells the nurse that the last food was buffalo chicken wings and beer. Which medical condition does the nurse find to be most consistent with the client's presenting problems?

Acute gastritis

dietary indiscretion—a person eats food that is irritating, too highly seasoned, or contaminated with disease-causing microorganisms. Other causes of acute gastritis include overuse of aspirin and other nonsteroidal anti-inflammatory drugs (NSAIDs), excessive alcohol intake, bile reflux, and radiation therapy.

Acute gastritis is often caused by

A physician has written an order for ranitidine (Zantac), 300 mg once daily. The nurse schedules the medication for which time?

At bedtime Currently, the most commonly used therapy for peptic ulcers is a combination of antibiotics, proton-pump inhibitors, and bismuth salts that suppress or eradicate H. pylori. Recommended therapy for 10 to 14 days includes triple therapy with two antibiotics (eg, metronidazole [Flagyl] or amoxicillin [Amoxil] and clarithromycin [Biaxin]) plus a proton-pump inhibitor (eg, lansoprazole [Prevacid], omeprazole [Prilosec], or rabeprazole [Aciphex]), or quadruple therapy with two antibiotics (metronidazole and tetracycline) plus a proton-pump inhibitor and bismuth salts (Pepto-Bismol). Research is being conducted to develop a vaccine against H. pylori.

While caring for a patient hospitalized with chronic gastritis it is important to educate the patient about what? A) How to eat nutritiously B) The correct use of medications to relieve acute gastritis C) How to position themselves to relieve pain D) Avoiding irritating foods and beverages

Avoiding irritating foods and beverages Measures to help relieve pain include instructing the patient to avoid foods and beverages that may be irritating to the gastric mucosa and instructing the patient about the correct use of medications to relieve chronic gastritis. Education for chronic gastritis does not routinely include how to eat nutritiously or positioning themselves to relieve pain. Since the patient has chronic gastritis, option B is incorrect.

The nurse is evaluating a client's ulcer symptoms to differentiate ulcer as duodenal or gastric. Which symptom should the nurse at attribute to a duodenal ulcer?

Awakening in pain The client with a gastric ulcer often awakens between 1 to 2 AM with pain, and ingestion of food brings relief. Vomiting is uncommon in clients with duodenal ulcer. Hemorrhage is less likely in clients with duodenal ulcer than those with gastric ulcer. The client with a duodenal ulcer may experience weight gain.

A client is scheduled for removal of the lower portion of the antrum of the stomach and a small portion of the duodenum and pylorus. What surgical procedure will the nurse prepare the client for?

Billroth I

removal of the lower portion of the antrum of the stomach (which contains the cells that secrete gastrin) as well as a small portion of the duodenum and pylorus. The remaining segment is anastomosed to the duodenum.

Billroth I procedure involves

The health care provider prescribes a combination of three drugs to treat peptic ulcer disease. The nurse, preparing to review the drug actions and side effects with the patient, understands that the triple combination should be in which order?

Bismuth salts, antibiotics, and proton pump inhibitors Refer to Table 23-1 in the text to review the recommended triple combination.

The nurse is assessing a client with advanced gastric cancer. The nurse anticipates that the assessment will reveal which finding?

Bloating after meals

The nurse is admitting a patient diagnosed with gastric cancer. The nurse is concerned that the patient's gastric cancer is progressing when the nurse assesses which symptom? A) Stomach pain relieved with antacids B) Anemia C) Diarrhea D) Bloating after meals

Bloating after meals The early symptoms of gastric cancer are often not definite. Some studies have shown that early symptoms, such as pain relieved with antacids, resemble those of benign ulcers. Symptoms of progressive disease include dyspepsia (indigestion), early satiety, weight loss, abdominal pain just above the umbilicus, loss or decrease in appetite, bloating after meals, nausea and vomiting, and symptoms similar to those of peptic ulcer disease.

nutritional deficits and may have cachexia. Therefore, correcting nutritional deficits is a top priority. Discharge planning before surgery is important, but correcting the nutritional deficits is a higher priority. Radiation therapy hasn't been proven effective for gastric cancer, and teaching about it preoperatively wouldn't be appropriate. Preventing DVT isn't a high priority before surgery, but it assumes greater importance after surgery.

Clients with gastric cancer commonly have

A morbidly obese client asks the nurse if medications are available to assist with weight loss. The nurse knows that the client would not be a candidate for phentermine if the following is part of the client's health history:

Coronary artery disease Phentermine, which requires a prescription, stimulates central noradrenergic receptors, causing appetite suppression. It may increase blood pressure and should not be taken by people with a history of heart disease, uncontrolled hypertension, hyperthyroidism, or glaucoma.

is an accronym for Dietary Approaches to Stop Hypertension.

DASH diet

Which of the following appears to be a significant factor in the development of gastric cancer?

Diet

Which of the following appears to be a significant factor in the development of gastric cancer? Diet Age Ethnicity Gender

Diet Diet seems to be a significant factor: a diet high in smoked, salted, or pickled foods and low in fruits and vegetables may increase the risk of gastric cancer. The typical patient with gastric cancer is between 50 and 70 years of age. Men have a higher incidence than women. Native Americans, Hispanic Americans, and African Americans are twice as likely as Caucasian Americans to develop gastric cancer.

Which of the following appears to be a significant factor in the development of gastric cancer?

Diet (Che's tortas)

is an unpleasant set of vasomotor and GI symptoms that occur in up to 76% of patients who have had bariatric surgery. Early symptoms include a sensation of fullness, weakness, faintness, dizziness, palpitations, diaphoresis, cramping pains, and diarrhea. These symptoms resolve once the intestine has been evacuated (i.e., with defecation).

Dumping syndrome

A client who had a Roux-en-Y bypass procedure for morbid obesity ate a chocolate chip cookie after a meal. After ingestion of the cookie, the client reported cramping pains, dizziness, and palpitation. After having a bowel movement, the symptoms resolved. What should the nurse educate the client about regarding this event? Gastric outlet obstruction Dumping syndrome Bile reflux Celiac disease

Dumping syndrome Dumping syndrome is an unpleasant set of vasomotor and GI symptoms that occur in up to 76% of patients who have had bariatric surgery. Early symptoms include a sensation of fullness, weakness, faintness, dizziness, palpitations, diaphoresis, cramping pains, and diarrhea. These symptoms resolve once the intestine has been evacuated (i.e., with defecation).

A 75-year-old Hispanic male is brought to the clinic by his daughter, who tells the nurse she doesn't know what is wrong with her father. The daughter states her father is confused, agitated, and restless and has been for several weeks. What is one diagnosis the nurse should suspect? A) Duodenal ulcers B) Gastric cancer C) Peptic ulcers D) Chronic gastritis

Gastric cancer Confusion, agitation, and restlessness may be the only symptoms seen in elderly patients, who may have no gastric symptoms until their tumors are well advanced. These are not generally symptoms of duodenal or peptic ulcers, which present with pain. These are also not generally symptoms of chronic gastritis.

Review the following four examples of ideal body weight (IBW), actual weight, and body mass index (BMI). Using three criteria for each example, select the body weight that indicates morbid obesity. IBW = 150 lbs; weight = 190 lbs; BMI = 26 kg/m2 IBW = 132 lbs; weight = 184 lbs; BMI = 28 kg/m2 IBW = 175 lbs; weight = 265 lbs; BMI = 29 kg/m2 IBW = 145 lbs; weight = 290 lbs; BMI = 31 kg/m2

IBW = 145 lbs; weight = 290 lbs; BMI = 31 kg/m2 The criteria for morbid obesity are a body weight that is twice IBW and a BMI that exceeds 30 kg/m2.

are nausea, vomiting, distended abdomen, and abdominal pain.

Indicators of pyloric obstruction

The client has been taking famotidine (Pepcid) at home. The nurse prepares a teaching plan for the client indicating that the medication acts primarily to achieve which of the following?

Inhibit gastric acid secretions.

refers to one that is hard to treat, relieve, or cure

Intractable ulcer

DRUG CLASS THAT CAUSE GASTRITIS

NSAIDS

An older adult patient had a gastrectomy performed several weeks ago and is being followed closely by the care team. Due to potential complications of this surgery, the nurse should closely monitor the patient's levels of: A. Prealbumin and bilirubin B. Creatinine and blood urea nitrogen (BUN) C. Ionized calcium and C-reactive protein D. Iron and vitamin B12

Iron and vitamin B12. Dietary deficiencies associated with gastrectomy include malabsorption of organic iron, which may require supplementation with oral or parenteral iron, and a low serum level of vitamin B12, which may require supplementation by the intramuscular route.

Which is a true statement regarding the nursing considerations in administration of metronidazole?

It leaves a metallic taste in the mouth. Metronidazole leaves a metallic taste in the mouth. It may cause anorexia and should be given with meals to decrease gastrointestinal upset. Metronidazole increases the blood-thinning effects of warfarin.

The nurse is teaching a client with peptic ulcer disease who has been prescribed misoprostol (Cytotec). What information from the nurse would be most accurate about misoprostol?

Prevents ulceration in clients taking nonsteroidal anti-inflammatory drugs (NSAIDs) Misoprostol is a synthetic prostaglandin that protects the gastric mucosa against ulceration and is used in clients who take NSAIDs. Misoprostol should be taken with food. It does not improve emptying of the stomach, and it increases (not decreases) mucus production.

Lansoprazole and esomeprazole

Proton Pump Inhibitors (PPIs)

A nurse practitioner, who is treating a patient with GERD, knows that responsiveness to this drug classification is validation of the disease. The drug classification is:

Proton pump inhibitors

also called gastric outlet obstruction (GOO), occurs when the area distal to the pyloric sphincter becomes scarred and stenosed from spasm or edema or from scar tissue that forms when an ulcer alternately heals and breaks down.

Pyloric obstruction

severe abdominal pain, rigid and tender abdomen, vomiting, elevated temperature, and increased heart rate.

Signs of penetration and perforation are

is a cytoprotective drug.

Sucralfate (Carafate)

Which of the following are signs/symptoms of perforation?

Sudden, severe upper abdominal pain

After teaching a client who has had a Roux-en-Y gastric bypass, which client statement indicates the need for additional teaching? A. "I need to chew my food slowly and thoroughly." B"I should pick cereals with less than 2 g of fiber per serving." C. "I need to drink 8 ounces of water before eating." D."A total serving should amount to be less than one cup.

"I need to drink 8 ounces of water before eating." After a Roux-en-Y gastric bypass, the client should not drink fluids with meals, withholding fluids for 15 minutes before eating to 90 minutes after eating.

A nurse is performing discharge teaching with a client who had a total gastrectomy. Which statement indicates the need for further teaching? "I'm going to visit my pastor weekly for a while." "I will have to take vitamin B12 shots up to 1 year after surgery." "I will call my physician if I begin to have abdominal pain." "I will weight myself each day and record the weight."

"I will have to take vitamin B12 shots up to 1 year after surgery." After a total gastrectomy, a client will need to take vitamin B12 shots for life. Dietary B12 is absorbed in the stomach, and the inability to absorb it could lead to pernicious anemia. Visiting clergy for emotional support is normal after receiving a cancer diagnosis. This action should be encouraged by the nurse. It's appropriate for the client to call the physician if he experiences signs and symptoms of intestinal blockage or obstruction, such as abdominal pain. Because a client with a total gastrectomy will receive enteral feedings or parenteral feedings, he should weigh himself each day and keep a record of the weights.

A patient has been brought to the emergency department with an arm injury. It is found that the patient has fractured the radius and may require surgery. The patient's last meal was at 6:00 PM, and the surgery is held for risk of a potential aspiration. What is the earliest time the patient could potentially have surgery? A) 12:00 AM B) 9:00 PM C) 8:00 AM D) 6:00 AM

12:00 AM Within four of eating, residual waste material passes into the terminal ileum. Therefore options B, C, and D are incorrect.

Early manifestations of dumping syndrome occur

15 to 30 minutes after eating.

A client with gastric cancer is having a resection. What is the nursing management priority for this client?

Correcting nutritional deficits

The PACU nurse calls report to the receiving nurse regarding a patient who is returning to the unit after a Billroth II (gastrojejunostomy). What complication will the receiving nurse assess this patient for related to this procedure? A) Feelings of hunger B) Constipation C) Feeling of fullness D) Gastric reflux

Feeling of fullness Following a Billroth II, the patient may have problems with feelings of fullness, dumping syndrome, and diarrhea. Hunger, constipation, and gastric reflux are not symptoms associated with this procedure.

The nurse is assessing a client with an ulcer for signs and symptoms of hemorrhage. The nurse interprets which condition as a sign/symptom of possible hemorrhage? Hematemesis Bradycardia Hypertension Polyuria

Hematemesis The nurse interprets hematemesis as a sign/symptom of possible hemorrhage from the ulcer. Other signs that can indicate hemorrhage include tachycardia, hypotension, and oliguria/anuria.

The nurse is cautiously assessing a client admitted with peptic ulcer disease because the most common complication that occurs in 10% to 20% of clients is: Hemorrhage Intractable ulcer Perforation Pyloric obstruction

Hemorrhage

Sudden, severe upper abdominal pain (persisting and increasing in intensity), which may be referred to the shoulders, especially the right shoulder, because of irritation of the phrenic nerve in the diaphragm; vomiting; collapse (fainting); extremely tender and rigid (boardlike) abdomen; and hypotension and tachycardia, indicating shock.

Signs and symptoms of perforation

A client is admitted to the hospital with an exacerbation of chronic gastritis. When assessing the client's nutritional status, the nurse should expect to find what type of deficiency? A.vitamin C B. vitamin B12 C. vitamin B6 D. vitamin A

vitamin B12

A client with a peptic ulcer is diagnosed with Helicobacter pylori infection. The nurse is teaching the client about the medications prescribed, including metronidazole, omeprazole, and clarithromycin. Which statement by the client indicates the best understanding of the medication regimen? A. "medications will kill the bacteria and stop the acid production." B. "I should take these medications only when I have pain from my ulcer." C. "My ulcer will heal because these medications will kill the bacteria." D. "These medications will coat the ulcer and decrease the acid production in my stomach."

"medications will kill the bacteria and stop the acid production."

A client has a family history of stomach cancer. Which of the following factors would further increase the client's risk for developing gastric cancer? Select all that apply. Caucasian ancestry Female gender Age 55 years High intake of fruits and vegetables Previous infection with H. pylori

-Age 55 years -Previous infection with H. pylori The typical client with gastric cancer is between 40 and 70 years, but gastric cancer can occur in younger people. Men have a higher incidence of gastric cancer than women. Native Americans, Hispanic Americans, and African Americans are twice as likely as Caucasian Americans to develop gastric cancer. A diet high in smoked, salted, or pickled foods and low in fruits and vegetables may increase the risk of gastric cancer. Other factors related to the incidence of gastric cancer include chronic inflammation of the stomach, H. pylori infection, pernicious anemia, smoking, achlorhydria, gastric ulcers, subtotal gastrectomy more than 20 years ago, and genetics.

A client with gastric cancer is having a resection. What is the nursing management priority for this client? Discharge planning Correcting nutritional deficits Preventing deep vein thrombosis (DVT) Teaching about radiation treatment

Correcting nutritional deficits Clients with gastric cancer commonly have nutritional deficits and may have cachexia. Therefore, correcting nutritional deficits is a top priority. Discharge planning before surgery is important, but correcting the nutritional deficits is a higher priority. Radiation therapy hasn't been proven effective for gastric cancer, and teaching about it preoperatively wouldn't be appropriate. Preventing DVT isn't a high priority before surgery, but it assumes greater importance after surgery.

A patient has been taking a 10-day course of antibiotics for pneumonia. The patient has been having white patches that look like milk curds in the mouth. What treatment will the nurse educate the patient about?

Nystatin (Mycostatin)

The nurse caring for a patient who has had bariatric surgery is developing a teaching plan for his patient. Which information listed below is essential to include? A) Drink a minimum of 90 mL of fluid with each meal. B) Eat six small meals daily spaced at equal intervals. C) Choose foods high in carbohydrates, especially simple sugars. D) Limit calories to no more than 1800 daily.

Eat six small meals daily spaced at equal intervals. Due to decreased stomach capacity, the patient must consume small meals at intervals to meet nutritional requirements while avoiding a feeling of fullness and complications such as dumping syndrome. The patient should not consume fluids with meals. This practice, as well as consumption of foods high in carbohydrates, contributes to the development of dumping syndrome. Recommended caloric intake following surgery is generally less than 1000 calories per day.

A patient comes to the clinic with the complaint, "I think I have an ulcer." What is a characteristic associated with peptic ulcer pain that the nurse should inquire about? Select all that apply.

Feeling of emptiness that precedes meals from 1 to 3 hours Severe gnawing pain that increases in severity as the day progresses Burning sensation localized in the back or mid-epigastrium

The nurse is developing a plan of care for a patient with peptic ulcer disease. What nursing interventions should be included in the care plan? Select all that apply.

Frequently monitoring hemoglobin and hematocrit levels Observing stools and vomitus for color, consistency, and volume Checking the blood pressure and pulse rate every 15 to 20 minutes

All postoperative patients are at risk for complications. In addition to these complications, what are patients undergoing gastric surgery at increased risk for? A) Dietary excess B) Bile obstruction C) Decreased intestinal motility D) Hemorrhage

Hemorrhage In addition to the complications to which all postoperative patients are subject, the patient undergoing gastric surgery is at increased risk for hemorrhage, dietary deficiencies, bile reflux, and dumping syndrome. These patients are not at increased risk for dietary excess, bile obstruction, or decreased intestinal motility.

Which of the following are characteristics associated with the Zollinger-Ellison syndrome (ZES)? Select all that apply. Constipation Hypocalcemia Severe peptic ulcers Extreme gastric hyperacidity Gastrin-secreting tumors of the pancreas

Severe peptic ulcers Extreme gastric hyperacidity Gastrin-secreting tumors of the pancreas

A patient you are caring for is going to have gastric surgery in the morning. It is important for you to assess the patient's nutritional status as well as assessing for the presence of bowel sounds. Why would you palpate the abdomen? A) To assess for Sister Joseph's nodule B) To assess for appendicitis C) To detect masses D) To detect rigidity

To detect masses The nurse assesses for the presence of bowel sounds and palpates the abdomen to detect masses or tenderness. The scenario does not indicate the patient has cancer, so the nurse would not be assessing for Sister Joseph's nodule. It would not be necessary to assess for appendicitis on a patient scheduled for gastric surgery. Rigidity of the abdomen is an indication of an emergent condition requiring surgery; you would not assess for rigidity in a patient already scheduled for gastric surgery.

After the return of bowel sounds and removal of the nasogastric tube, the nurse may give fluids, followed by food in small portions. Foods are gradually added until the client can eat six small meals a day and drink 120 mL of fluid between meals.

recovering from gastric surgery

include alcohol abuse, smoking, and stress. A sedentary lifestyle and a history of hemorrhoids aren't risk factors for peptic ulcers.

risk factors for peptic (gastric and duodenal) ulcers

A client is prescribed tetracycline to treat peptic ulcer disease. Which instruction would the nurse give the client? "Take the medication with milk." "Be sure to wear sunscreen while taking this medicine." "Expect a metallic taste when taking this medicine, which is normal." "Do not drive when taking this medication."

"Be sure to wear sunscreen while taking this medicine." Tetracycline may cause a photosensitivity reaction in clients. The nurse should caution the client to use sunscreen when taking this drug. Dairy products can reduce the effectiveness of tetracycline, so the nurse should not advise him or her to take the medication with milk. A metallic taste accompanies administration of metronidazole (Flagyl). Administration of tetracycline does not necessitate driving restrictions.

A 70-year-old woman with a complex medical history made an appointment with her primary care provider because she has recently been experiencing heartburn, abdominal pain, and nausea. The clinician has identified that the woman's symptoms are characteristic of acute gastritis. Which of the woman's following statements is suggestive of the etiology of her problem? A. "I remember my father often complaining about heartburn and indigestion." B. "My endocrinologist recently increased my dose of metformin that I take for my diabetes." C. "I've been taking glucosamine supplements because I've been told they'll help my arthritis." D. "I've changed from taking Tylenol for my arthritis pain to taking aspirin."

"I've changed from taking Tylenol for my arthritis pain to taking aspirin."

A patient has been diagnosed with acute gastritis and asks the nurse what could have caused it. What is the best response by the nurse? (Select all that apply.) "It can be caused by ingestion of strong acids." "You may have ingested some irritating foods." "Is it possible that you are overusing aspirin." "It is a hereditary disease." "It is probably your nerves."

"It can be caused by ingestion of strong acids." "You may have ingested some irritating foods." "Is it possible that you are overusing aspirin." Acute gastritis is often caused by dietary indiscretion—the person eats food that is irritating, too highly seasoned, or contaminated with disease-causing microorganisms. Other causes of acute gastritis include overuse of aspirin and other nonsteroidal anti-inflammatory drugs (NSAIDs), excessive alcohol intake, bile reflux, and radiation therapy. A more severe form of acute gastritis is caused by the ingestion of strong acid or alkali, which may cause the mucosa to become term-31gangrenous or to perforate.

You are doing patient teaching with a patient has just been prescribed Prevacid. What statement would indicate that the patient correctly understands the action of this medication? A) "The medication inhibits acid secretions." B) "The medication is an antibiotic." C) "The medication is an analgesic." D) "The medication will repair my ulcer."

"The medication inhibits acid secretions." The statement, "The medication inhibits acid secretions," indicates that the patient understands that the medication inhibits acid secretion. This medication does not act as an antibiotic or analgesic, nor will it repair the ulcer.

A patient with a diagnosis of peptic ulcer disease has just been prescribed omeprazole (Prilosec). How should the nurse best describe this medication's therapeutic action? 1. "This medication will reduce the amount of acid secreted in your stomach." 2. "This medication will make the lining of your stomach more resistant to damage." 3. "This medication will specifically address the pain that accompanies peptic ulcer disease." 4. "This medication will help your stomach lining to repair itself."

"This medication will reduce the amount of acid secreted in your stomach." Proton pump inhibitors like Prilosec inhibit the synthesis of stomach acid. PPIs do not increase the durability of the stomach lining, relieve pain, or stimulate tissue repair.

A client with acute gastritis asks the nurse what might have caused the problem. Which of the following are possible causes?

-Dietary indiscretion -Excessive alcohol intake -Radiation therapy Possible causes of gastritis include dietary indiscretion, overuse of aspirin and other nonsteroidal anti-inflammatory drugs, excessive alcohol intake, bile reflux, and radiation therapy. Allergy medicine and fruit juices are not causes of acute gastritis.

The nurse is caring for a client who has just returned from the PACU after surgery for peptic ulcer disease. For what potential complications does the nurse know to monitor? Select all that apply.

-Hemorrhage -Perforation -Penetration -Pyloric obstruction Potential complications may include hemorrhage, perforation, penetration, and pyloric obstruction. A client who has had surgery for peptic ulcer disease may have a decreased appetite in the immediate postoperative stage, but it is not something the nurse would monitor for and would not cause cachexia. Inability to clear secretions is generally not a complication of peptic ulcer surgery.

Which of the following interventions are appropriate for clients with gastritis? Select all that apply.

-Use a calm approach to reduce anxiety. -Discourage cigarette smoking. -Notify the physician of inidicators of hemorrhagic gastritis. The nurse should use a calm approach when answering questions and providing teaching. He or she should discuss smoking cessation and monitor for any indicators of hemorrhagic gastritis. The client will take nothing by mouth for up to a few days until symptoms subside. The nurse needs to develop an individualized teaching plan for the client that includes information about stress management, diet, and medications.

A patient has had a gastrostomy tube inserted. What does the nurse anticipate the initial fluid nourishment will be after the insertion of the gastrostomy tube?

10% glucose and tap water

A nursing student is preparing a teaching plan about peptic ulcer disease. The student knows to include teaching about the percentage of clients with peptic ulcers who experience bleeding. The percentage is

15% Fifteen percent of clients with peptic ulcer experience bleeding.

A patient sustained second- and third-degree burns over 30% of the body surface area approximately 72 hours ago. What type of ulcer should the nurse be alert for while caring for this patient?

Curling's ulcer

A patient sustained second- and third-degree burns over 30% of the body surface area approximately 72 hours ago. What type of ulcer should the nurse be alert for while caring for this patient? Curling's ulcer Peptic ulcer Esophageal ulcer Meckel's ulcer

Curling's ulcer

A client reports to the clinic, stating that she rapidly developed headache, abdominal pain, nausea, hiccuping, and fatigue about 2 hours ago. For dinner, she ate buffalo chicken wings and beer. Which of the following medical conditions is most consistent with the client's presenting problems? Acute gastritis Duodenal ulcer Gastric cancer Gastric ulcer

Acute gastritis A client with acute gastritis may have a rapid onset of symptoms, including abdominal discomfort, headache, lassitude, nausea, anorexia, vomiting, and hiccupping, which can last from a few hours to a few days. Acute gastritis is often caused by dietary indiscretion-a person eats food that is irritating, too highly seasoned, or contaminated with disease-causing microorganisms. A client with a duodenal ulcer will present with heartburn, nausea, excessive gas and vomiting. A client with gastric cancer will have persistent symptoms of nausea and vomiting, not sudden symptoms. A client with a gastric ulcer will have bloating, nausea, and vomiting, but not necessarily hiccups.

The nurse is assessing an 80-year-old client for signs and symptoms of gastric cancer. The nurse differentiates which as a sign/symptom of gastric cancer in the geriatric client, but not in a client under the age of 75?

Agitation The nurse understands that agitation, along with confusion and restlessness, may be the only signs/symptoms seen of gastric cancer in the older client. Abdominal mass, hepatomegaly, and ascites may all be signs/symptoms of advanced gastric cancer.

A client experienced extensive burns and 72 hours later has developed an ulcer. Which of the following types of ulcer is most likely in this client?

Curling's ulcer Curling's ulcer is frequently observed about 72 hours after extensive burns and involves the antrum of the stomach or duodenum.

What is currently the most commonly used therapy for peptic ulcers? A) Bismuth salts, antibiotics, and histamine-2 antagonists B) H2 antagonists, antibiotics, and bicarbonate salts C) Bicarbonate salts, antibiotics, and ZES D) Antibiotics, proton pump inhibitors, and bismuth salts

Antibiotics, proton pump inhibitors, and bismuth salts Currently, the most commonly used therapy for peptic ulcers is a combination of antibiotics, proton pump inhibitors, and bismuth salts that suppress or eradicate H. pylori. H2 receptor antagonists are used to treat NSAID-induced ulcers and other ulcers not associated with H. pylori infection, but they are not the drug of choice; proton pump inhibitors are. Bicarbonate salts are not used. ZES is the Zollinger-Ellison syndrome and not a drug.

A client with gastric cancer is scheduled to undergo a Billroth II procedure. The client's spouse asks how much of the client's stomach will be removed. Which of the following would be the most accurate response from the nurse?

Approximately 75% The Billroth II is a wide resection that involves removing approximately 75% of the stomach and decreases the possibility of lymph node spread or metastatic recurrence.

While caring for a patient who has had radical neck surgery, the nurse notices an abnormal amount of serosanguineous secretions in the wound suction unit during the first postoperative day. What does the nurse know is an expected amount of drainage in the wound unit?

Approximately 80 to 120 mL

Which ulcer is associated with extensive burn injury? Cushing ulcer Curling ulcer Peptic ulcer Duodenal ulcer

Curling ulcer

A nurse caring for a patient in a burn treatment center knows to assess for the presence of which of the following types of ulcer about 72 hours post injury? Curling's ulcer Peptic ulcer Esophageal ulcer Meckel's ulcer

Curling's ulcer Curling's ulcer is frequently observed about 72 hours after extensive burns and involves the antrum of the stomach or the duodenum. Peptic, esophageal, and Meckel's ulcers are not related to burn injuries.term-13

A client sustained second- and third-degree burns over 30% of the body surface area approximately 72 hours ago. What type of ulcer should the nurse be alert for while caring for this client? Curling's ulcer Peptic ulcer Esophageal ulcer Meckel's ulcer

Curling's ulcer Curling's ulcer is frequently observed about 72 hours after extensive burns and involves the antrum of the stomach or the duodenum. Peptic, esophageal, and Meckel's ulcers are not related to burn injuries.

A patient sustained second- and third-degree burns over 30% of the body surface area approximately 72 hours ago. What type of ulcer should the nurse be alert for while caring for this patient? A. Curling's ulcer B. Meckel's ulcer C. Peptic ulcer D. Esophageal ulcer

Curling's ulcer Ulcer is associated with burns

A patient is complaining of diarrhea after having bariatric surgery. What nonpharmacologic treatment can the nurse suggest to decrease the incidence of diarrhea?

Decrease the fat content in the diet. Patients may complain of either diarrhea or constipation postprocedure. Diarrhea is more common an occurrence post bariatric surgery, particularly after malabsorptive procedures (Mechanick et al., 2008). Both may be prevented if the patient consumes a nutritious diet that is high in fiber. Steatorrhea also may occur as a result of rapid gastric emptying, which prevents adequate mixing with pancreatic and biliary secretions. In mild cases, reducing the intake of fat and administering an antimotility medication (e.g., loperamide [Imodium]) may control symptoms.

Which of the following are clinical manifestations of gastroesophageal reflux disease (GERD)? Select all that apply.

Regurgitation Dyspepsia Pyrosis Hypersalivation Esophagitis

When preparing a patient who has had gastric surgery to go home, what information is given to the patient, significant other, and family? A) Location of local senior center B) Locations of elder care facilities C) End-of-life care, when indicated D) Contact number for Meals-on-Wheels

End-of-life care, when indicated Information about community support groups and end-of-life care is provided to the patient, family, or significant other when indicated. The location of the local senior center, elder care facilities, or the contact number for Meals-on-Wheels would probably not be appropriate for all patients as many are under the age of 55.

A health care provider suspects that a client has peptic ulcer disease. With which diagnostic procedure would the nurse most likely prepare to assist? Barium study of the upper gastrointestinal tract Endoscopy Gastric secretion study Stool antigen test

Endoscopy

A client has been taking famotidine at home. What teaching should the nurse include with the client?

Famotidine will inhibit gastric acid secretions.

What are the complications that can occur following gastric surgery? (Mark all that apply.) A) Dumping syndrome B) Stomal ulcers C) Pneumonia D) Metabolic imbalances E) Gastritis

Stomal ulcers C) Pneumonia D) Metabolic imbalances Complications that may occur in the immediate postoperative period include peritonitis, stomal obstruction, stomal ulcers, atelectasis and pneumonia, thromboembolism, and metabolic imbalances resulting from prolonged vomiting and diarrhea or altered gastrointestinal function. Dumping syndrome does not generally occur in the immediate postoperative period, and neither does gastritis.

A nurse is caring for a client hospitalized with an exacerbation of chronic gastritis. What health promotion topic should the nurse emphasize? 1. Strategies for maintaining an alkaline gastric environment 2. A safe technique for self-suctioning 3. Techniques for positioning correctly to promote gastric healing 4. Strategies for avoiding irritating foods and beverages

Strategies for avoiding irritating foods and beverages Measures to help relieve pain include instructing the client to avoid foods and beverages that may be irritating to the gastric mucosa and instructing the client about the correct use of medications to relieve chronic gastritis. An alkaline gastric environment is neither possible nor desirable. There is no plausible need for self-suctioning. Positioning does not have a significant effect on the presence or absence of gastric healing.

including abdominal discomfort, headache, lassitude, nausea, anorexia, vomiting, and hiccuping, which can last from a few hours to a few days. Often caused by dietary indiscretion—a person eats food that is irritating, too highly seasoned, or contaminated with disease-causing microorganisms.

acute gastritis may have a rapid onset of symptoms,

A patient asks the home health nurse from what the distressing symptoms of dumping syndrome result. What physiological occurrence should the nurse explain? A. Reflux of bile into the distal esophagus b. B. Irritation of the phrenic nerve due to diaphragmatic pressure C. Chronic malabsorption of iron and vitamins A and C D .Osmotic transport of extracellular fluid into the gastrointestinal tract

Osmotic transport of extracellular fluid into the gastrointestinal tract

Which of the following is considered an early symptom of gastric cancer?

Pain relieved by antacids Symptoms of early disease, such as pain relieved by antacids, resemble those of benign ulcers and are seldom definitive. Symptoms of progressive disease include weight loss, bloating after meals, and dyspepsia.

A 30-year-old obese female patient who underwent gastric banding 3 days ago is getting ready to go home. Essential postoperative teaching for this client should include instruction related to the importance of abstaining from what for the next 2 years? A) Multivitamin supplements B) Pregnancy C) Antidepressants D) Control top panty hose

Pregnancy Women of childbearing age who have had bariatric surgery should avoid pregnancy for approximately 2 years until their weight stabilizes, and it is evident that their nutritional needs are being adequately met. Multivitamins are generally recommended for the client to supplement dietary sources of nutrients. Antidepressants may be taken if clinically indicated. Control top pantyhose may be uncomfortable postoperatively; however, they are not contraindicated.

People with blood type O are

more susceptible to peptic ulcers than those with blood type A, B, or AB.

The healthcare provider of a client with oral cancer has ordered the placement of a GI tube to provide nutrition and to deliver medications. What would be the preferred route

nasogastric intubation

is frequently observed about 72 hours after extensive burns and involves the antrum of the stomach or the duodenum.

Curling ulcer

A nurse caring for a patient in a burn treatment center knows to assess for the presence of which of the following types of ulcer about 72 hours post injury?

Curling's

A nurse is teaching a client with gastritis about the need to avoid the intake of caffeinated beverages. The client asks why this is so important. Which explanation from the nurse would be most accurate? "Caffeine stimulates the central nervous system and thus gastric activity and secretions, which need to be minimized to promote recovery." "Caffeine increases the fluid volume in your system, which irritates your digestive organs." "Caffeine intake can cause tears in your esophagus and intestines, which can lead to hemorrhage." "Caffeine can interfere with absorption of vitamin B12, which leads to anemia and further digestive problems."

"Caffeine stimulates the central nervous system and thus gastric activity and secretions, which need to be minimized to promote recovery." Caffeine is a central nervous system stimulant that increases gastric activity and pepsin secretion. Caffeine is a diuretic that causes decreased fluid volume and potential dehydration. It does not lead to hemorrhage and does not interfere with absorption of vitamin B12.

The nurse determines that teaching for the client with peptic ulcer disease has been effective when the client makes which statement? "I should stop all my medications if I develop any side effects." "I should continue my treatment regimen as long as I have pain." "I have learned some relaxation strategies that decrease my stress." "I can buy whatever antacids are on sale because they all have the same effect."

"I have learned some relaxation strategies that decrease my stress." The nurse assists the client to identify stressful or exhausting situations. A hectic lifestyle and an irregular schedule may aggravate symptoms and interfere with regular meals taken in relaxed settings along with the regular administration of medications. The client may benefit from regular rest periods during the day, at least during the acute phase of the disease. Biofeedback, hypnosis, behavior modification, massage, or acupuncture may be helpful.

A client with gastric cancer is having a resection. What is the nursing management priority for this client? A. Preventing deep vein thrombosis (DVT) B. Teaching about radiation treatment C. Discharge planning D. Correcting nutritional deficits

Correcting nutritional deficits Clients with gastric cancer commonly have nutritional deficits and may have cachexia . Therefore, correcting nutritional deficits is a top priority. Chachexia= wasting syndrome.A general state of ill health involving marked weight loss and muscle loss

A patient comes to the bariatric clinic to obtain information about bariatric surgery. The nurse assesses the obese patient knowing that in addition to meeting the criterion of morbid obesity, a candidate for bariatric surgery must also demonstrate what? A) Knowledge of the causes of obesity and its associated risks B) Emotional stability and understanding of required lifestyle changes C) Positive body image and high self-esteem D) Insight into why their past weight loss efforts failed

Emotional stability and understanding of required lifestyle changes Patients seeking bariatric surgery must be evaluated by a psychiatrist, psychologist, or advanced practice mental health nurse to establish that they are free of serious mental disorders, and are motivated to comply with lifestyle changes related to eating patterns, dietary choices, and elimination. Obese patients are often unlikely to have a positive body image due to the social stigma associated with obesity. While assessment of knowledge about causes of obesity and its associated risks as well as insight into the reasons why previous diets have been ineffective are included in the client's plan of care, these do not predict positive client outcomes following bariatric surgery. Most obese patients have an impaired body image and alteration in self-esteem. An obese patient with a positive body image would be unlikely to seek this surgery unless he or she was experiencing significant comorbidities.

As a nurse completes the admission assessment of a client admitted for gastric bypass surgery, the client states, "Finally! I'll be thin and able to eat without much concern." How should the nurse intervene?

Evaluate the client's understanding of the procedure.

Familial tendency also may be a significant predisposing factor. People with blood type O are more susceptible to peptic ulcers than are those with blood type A, B, or AB. Blood type is not a predisposing factor for gastric cancer, esophageal varices, and diverticulitis.

GI disorders

Diet seems to be a significant factor: a diet high in smoked, salted, or pickled foods and low in fruits and vegetables may increase the risk .The typical patient is between 50 and 70 years of age. Men have a higher incidence than women. Native Americans, Hispanic Americans, and African Americans are twice as likely as Caucasian Americans to develop

Gastric cancer

A patient comes to the clinic for a routine checkup. During the assessment the patient states that they have had pain in the mid-epigastric region off and on for "several weeks." The nurse suspects a peptic ulcer and asks the patient to describe their pain. A patient with a peptic ulcer usually describes the pain as what? A) Gnawing B) Sharp stabbing C) Overdistended feeling D) Cramping or aching

Gnawing Patients with ulcers generally describe the pain as a gnawing or dull pain. They may also complain of a burning sensation. Patients do not generally describe their pain as "sharp stabbing," an "overdistended feeling," or as "cramping or aching."

Which diagnostic test would be used first to evaluate a client with upper GI bleeding?

Hemoglobin and hematocrit The nurse assesses for faintness or dizziness and nausea, which may precede or accompany bleeding. It is important to monitor vital signs frequently and to evaluate for tachycardia, hypotension, and tachypnea. Other nursing interventions include monitoring the hemoglobin and hematocrit, testing the stool for gross or occult blood, and recording hourly urinary output to detect anuria or oliguria (absence of or decreased urine production). If bleeding cannot be managed by the measures described, other treatment modalities such as endoscopy may be used to halt bleeding and avoid surgical intervention. There is debate regarding how soon endoscopy should be performed. Some clinicians believe endoscopy should be performed within the first 24 hours after hemorrhaging has ceased. Others believe endoscopy may be performed during acute bleeding, as long as the esophageal or gastric area can be visualized (blood may decrease visibility). An upper GI is less accurate than endoscopy and would not reveal a bleed. Arteriography is an invasive study associated with life-threatening complications and would not be used for an initial evaluation.

The nurse is caring for a patient who is scheduled for a colonoscopy. The nurse is preparing to instruct the patient on a colon preparation procedure that will include polyethylene glycol electrolyte lavage prior to the procedure. What is the nurse aware of about the use of lavage solutions and when they are contraindicated? A) In a patient with an inflammatory bowel disease B) In a patient with polyps C) In a patient with a colostomy D) In a patient with colon cancer

In a patient with an inflammatory bowel disease The use of a lavage solution is contraindicated in patients with intestinal obstruction or inflammatory bowel disease. Patients with a colostomy can receive a lavage solution. Cleansing of the colon using polyethylene glycol electrolyte lavage is a common procedure that precedes a colonoscopy, as a colonoscopy is the most frequently used diagnostic aid and screening device for patients with previous colon cancer or polyps.

The nurse is assessing a patient on antibiotic therapy for gastritis. The patient requests more information about the typical causes of Helicobacter pylori infection. What would it be appropriate for the nurse to instruct the patient? A) Most affected patients acquired the infection during international travel. B) Infection typically occurs due to ingestion of contaminated food and water. C) A genetic factor predisposing individuals to H. pylori infection is having type A blood. D) Person-to-person transmission of the H. pylori organism does not occur.

Infection typically occurs due to ingestion of contaminated food and water. Acute gastritis is often caused by dietary indiscretion—a person eats food that is irritating, too highly seasoned, or contaminated with disease-causing microorganisms. A predisposing factor for acquiring H. pylori infection is having type O blood. The organism is endemic to the United States. Person-to-person transmission is possible through contact with saliva or emesis.

A patient presents to the walk-in clinic complaining of vomiting and burning in their mid-epigastria. The nurse knows that to confirm peptic ulcer disease, the physician is likely to order a diagnostic test to detect the presence of what? A) Infection with Helicobacter pylori B) Excessive stomach acid secretion C) Gastric irritation caused by NSAIDs D) Over-consumption of spicy foods

Infection with Helicobacter pylori H. pylori infection may be determined by endoscopy and histologic examination of a tissue specimen obtained by biopsy, or a rapid urease test of the biopsy specimen. Other less invasive diagnostic measures for detecting H. pylori include serologic testing for antibodies against the H. pylori antigen, stool antigen test, and urea breath test. Excessive stomach acid secretion, NSAIDs, and dietary indiscretion may all cause gastritis; however, peptic ulcers are caused by colonization of the stomach by H. pylori.

A nursing student is caring for a client with gastritis. Which of the following would the student recognize as a common cause of gastritis? Choose all that apply. Ingestion of strong acids Irritating foods Overuse of aspirin DASH diet Participation in highly competitive sports

Ingestion of strong acids Irritating foods Overuse of aspirin Acute gastritis is often caused by dietary indiscretion-a person eats food that is irritating, too highly seasoned, or contaminated with disease-causing microorganisms. Other causes of acute gastritis include overuse of aspirin and other nonsteroidal anti-inflammatory drugs (NSAIDs), excessive alcohol intake, bile reflux, and radiation therapy. A more severe form of acute gastritis is caused by the ingestion of strong acid or alkali, which may cause the mucosa to become gangrenous or to perforate. A DASH diet is an acronym for Dietary Approaches to Stop Hypertension, which would not cause gastritis. Participation in competitive sports also would not cause gastritis.

The nurse is managing a gastric (Salem) sump tube for a patient who has an intestinal obstruction and will be going to surgery. What interventions should the nurse perform to make sure the tube is functioning properly?

Keep the vent lumen above the patient's waist to prevent gastric content reflux.

A 66-year-old African-American client has recently visited a physician to confirm a diagnosis of gastric cancer. The client has a history of tobacco use and was diagnosed 10 years ago with pernicious anemia. He and his family are shocked about the possibility of cancer because he was asymptomatic prior to recent complaints of pain and multiple gastrointestinal symptoms. On the basis of knowledge of disease progression, the nurse assumes that organs adjacent to the stomach are also affected. Which of the following organs may be affected? Choose all that apply. Liver Pancreas Bladder Duodenum Lungs

Liver Pancreas Duodenum

Rebleeding may occur from a peptic ulcer and often warrants surgical interventions. Signs of bleeding include which of the following? A. Bradycardia B. Bradypnea C. Mental confusion D. Hypertension

Mental confusion Signs of bleeding include tachycardia, tachypnea, hypotension, mental confusion, thirst, and oliguria.

Which of the following medications, used in the treatment of GERD, accelerate gastric emptying

Metoclopramide (Reglan)

is an antibiotic, specifically an amebicide.

Metronidazole

The Home Health Nurse is preparing for an initial home visit to a patient discharged following a total gastrectomy for treatment of gastric cancer. What would the nurse anticipate that the plan of care is most likely to include? A) Encouraging the patient to eat small, high-fiber meals at frequent intervals B) Gastrointestinal decompression by nasogastric tube C) Periodic assessment for esophageal distension D) Monthly administration of injections of vitamin B

Monthly administration of injections of vitamin B Since vitamin B is absorbed in the stomach, the patient requires vitamin B replacement to prevent pernicious anemia. The patient who has had a total gastrectomy is likely to require parenteral nutrition. Since the stomach is absent, a nasogastric tube would not be indicated. Since there is no stomach to act as a reservoir and fluids and nutrients are passing directly into the jejunum, distension is unlikely.

Which is a true statement regarding gastric cancer? Most clients are asymptomatic during the early stage of the disease. Women have a higher incidence of gastric cancer. The prognosis for gastric cancer is good. Most cases are discovered before metastasis.

Most clients are asymptomatic during the early stage of the disease. Most clients are asymptomatic during the early stage of the disease. Men have a higher incidence of gastric cancer. The prognosis is poor because the diagnosis is usually made late because most clients are asymptomatic during the early stage. Most cases of gastric cancer are discovered only after local invasion has advanced or metastases are present.

can occur anywhere in the stomach. The tumor infiltrates the surrounding mucosa, penetrating the wall of the stomach and adjacent organs and structures. The liver, pancreas, esophagus, and duodenum are often already affected at the time of diagnosis. Metastasis through lymph to the peritoneal cavity occurs later in the disease.

Most gastric cancers are adenocarcinomas

The nurse in the ED admits a client with suspected gastric outlet obstruction. The client's symptoms include nausea and vomiting. The nurse anticipates that the physician will issue which order?

Nasogastric tube insertion The nurse anticipates an order for nasogastric tube insertion to decompress the stomach. Pelvic x-ray, oral contrast, and stool specimens are not indicated at this time.

A nurse is caring for a client who has a diagnosis of GI bleed. During shift assessment, the nurse finds the client to be tachycardic and hypotensive, and the client has an episode of hematemesis while the nurse is in the room. In addition to monitoring the client's vital signs and level of conscious, what would be a priority nursing action for this client? 1. Place the client in a prone position. 2. Provide the client with ice water to slow any GI bleeding. 3. Prepare for the insertion of an NG tube. 4. Notify the health care provider.

Notify the health care provider. The nurse must always be alert for any indicators of hemorrhagic gastritis, which include hematemesis (vomiting of blood), tachycardia, and hypotension. If these occur, the physician is notified and the client's vital signs are monitored as the client's condition warrants. Putting the client in a prone position could lead to aspiration. Giving ice water is contraindicated as it would stimulate more vomiting.

You are caring for a patient who has a diagnosis of GI bleed. When you make your shift assessment, you find the patient is tachycardic and hypotensive. The patient has an episode of hematemesis while you are in the room. What would be a priority nursing action for this patient? A) Put them in a high Fowler's position. B) Give them ice water. C) Assess for pain. D) Notify the physician.

Notify the physician. The nurse must always be alert for any indicators of hemorrhagic gastritis, which include hematemesis (vomiting of blood), tachycardia, and hypotension. If these occur, the physician is notified and the patient's vital signs are monitored as the patient's condition warrants. Putting the patient in a high Fowler's position is not a priority and neither is assessing for pain. Giving them ice water is contraindicated as it would stimulate more vomiting.

A client undergoes total gastrectomy. Several hours after surgery, the nurse notes that the client's nasogastric (NG) tube has stopped draining. How should the nurse respond?

Notify the physician. The nurse should notify the physician because an NG tube that fails to drain during the postoperative period may be clogged, which could increase pressure on the suture site because fluid isn't draining adequately. Repositioning or irrigating an NG tube in a client who has undergone gastric surgery can disrupt the anastomosis. Increasing the level of suction may cause trauma to GI mucosa or the suture line.

The nurse is developing a plan of care for a patient with peptic ulcer disease. What nursing interventions should be included in the care plan? Select all that apply.

Observing stools and vomitus for color, consistency, and volume Checking the blood pressure and pulse rate every 15 to 20 minutes Frequently monitoring hemoglobin and hematocrit levels

Which medication classification represents a proton (gastric acid) pump inhibitor

Omeprazole

An important part of the nurse's assessment of the patient with peptic ulcer disease is assessing the patient's level of anxiety. How is this accomplished? A) Appropriate information is provided to the patient's family. B) Patient is encouraged to express fears openly. C) Some patients aren't anxious so this assessment isn't necessary. D) Information is given in written form.

Patient is encouraged to express fears openly. The nurse assesses the patient's level of anxiety. Patients with peptic ulcers are usually anxious, but their anxiety is not always obvious. Appropriate information is provided at the patient's level of understanding, all questions are answered, and the patient is encouraged to express fears openly.

The dietitian is giving an inservice to the nurses on your unit. Many of your elderly patients experience poor wound healing after surgery. The dietitian has determined that these patients require more protein in their diet. The nurses are aware that enzymes are essential in the digestion of nutrients. What is the enzyme that initiates the digestion of protein? A) Pepsin B) Intrinsic factor C) Lipase D) Amylase

Pepsin The enzyme that initiates the digestion of protein is pepsin. Intrinsic factor combines with vitamin B12 for absorption by the ileum. Amylase aids in the digestion of starch, and lipase aids in the digestion of fats.

The nurse is conducting a community education class on gastritis. The nurse includes that chronic gastritis caused by Helicobacter pylori is implicated in which disease/condition? Pernicious anemia Systemic infection Peptic ulcers Colostomy

Peptic ulcers Chronic gastritis caused by Helicobacter pylori is implicated in the development of peptic ulcers. Chronic gastritis is sometimes associated with autoimmune disease, such as pernicious anemia, but not as a cause of the anemia. Chronic gastritis is not implicated in system infections and/or colostomies.

gastroduodenal mucosa because this tissue cannot withstand the digestive action of gastric acid (HCl) and pepsin.

Peptic ulcers occur mainly in the

is erosion of the ulcer through the gastric serosa into the peritoneal cavity without warning.

Perforation

A patient is in the hospital for the treatment of peptic ulcer disease. The nurse finds the patient vomiting and complaining of a sudden severe pain in the abdomen. The nurse then assesses a board-like abdomen. What does the nurse suspect these symptoms indicate?

Perforation of the peptic ulcer Signs and symptoms of perforation include the following: Sudden, severe upper abdominal pain (persisting and increasing in intensity), which may be referred to the shoulders, especially the right shoulder, because of irritation of the phrenic nerve in the diaphragm; vomiting; collapse (fainting); extremely tender and rigid (boardlike) abdomen; and hypotension and tachycardia, indicating shock.

The nurse is providing patient education for a patient with peptic ulcer disease secondary to chronic nonsteroidal anti-inflammatory drug (NSAID) use. The patient has been newly prescribed misoprostol (Cytotec). What would the nurse be most accurate in informing the patient about the drug? A) Reduces the stomach's volume of hydrochloric acid B) Increases the speed of gastric emptying C) Protects the stomach's lining D) Increases lower esophageal sphincter pressure

Protects the stomach's lining Misoprostol is a synthetic prostaglandin that, like prostaglandin, protects the gastric mucosa. NSAIDs decrease prostaglandin production and predispose the patient to peptic ulceration. Misoprostol doesn't reduce gastric acidity (option A), improve emptying of the stomach (option B), or increase lower esophageal sphincter pressure (option D).

A nurse is preparing to discharge a client newly diagnosed with peptic ulcer disease. The client's diagnostic test results were positive for H. pylori bacteria. The doctor has ordered the "triple therapy" regimen. Which of the following is the correct representation of "triple therapy" refers?

Proton-pump inhibitor and two antibiotics Currently, the most commonly used therapy for peptic ulcers is a combination of antibiotics, proton-pump inhibitors, and bismuth salts that suppress or eradicate H. pylori bacteria. Recommended therapy for 10 to 14 days includes triple therapy with two antibiotics (eg, metronidazole [Flagyl] or amoxicillin [Amoxil] and clarithromycin [Biaxin]) plus a proton-pump inhibitor (eg, lansoprazole [Prevacid], omeprazole [Prilosec], or rabeprazole [Aciphex]), or quadruple therapy with two antibiotics (metronidazole and tetracycline) plus a proton-pump inhibitor and bismuth salts (Pepto-Bismol). Research is being conducted to develop a vaccine against H. pylori.

The nurse is caring for a patient who has just returned from the PACU after having surgery for peptic ulcer disease. What potential complication does the nurse know to monitor for? A) Decreased appetite B) Pyloric obstruction C) Increased salivation D) Inability to clear secretions

Pyloric obstruction Potential complications may include the following: hemorrhage, perforation, penetration, and pyloric obstruction (gastric outlet obstruction). A patient who has had surgery for peptic ulcer disease may have a decreased appetite in the immediate postoperative stage, but it is not something the nurse would monitor for. Increased salivation and inability to clear secretions are generally not complications of peptic ulcer surgery.

when the area distal to the pyloric sphincter becomes scarred and stenosed from spasm or edema or from scar tissue that forms when an ulcer alternately heals and breaks down.

Pyloric obstruction, also called gastric outlet obstruction (GOO), occurs

Josephine Hites, an 80-year-old retired dress shop clerk, is reporting for her semiannual physical at the primary care group where you practice nursing. She demonstrates an unintentional loss of ten pounds in the last six months. When asked, she reports "Food just doesn't taste good anymore!" If her anorexia becomes chronic, what effect could this have on her blood cell counts?

RBCs increase in size

Which of the following is the most successful treatment for gastric cancer? Palliation Radiation Chemotherapy Removal of the tumor

Removal of the tumor There is no successful treatment for gastric carcinoma except removal of the tumor. If the tumor can be removed while it is still localized to the stomach, the patient may be cured. If the tumor has spread beyond the area that can be excised, cure is less likely.

Which of the following medications used for obesity improves cardiovascular disease risk factors in obese patients with metabolic syndrome?

Rimonabant (Acomplia) Acomplia is the newest medication used to treat obesity. It stimulates weight reduction and improves cardiovascular disease risk factors in obese patients with metabolic syndrome. Meridia was recently pulled from the market because of the increased risk of heart attack and stroke associated with this medication. Orlistat, available by prescription and over the counter as Alli, reduces caloric intake by binding to gastric and pancreatic lipase to prevent digestion of fats.

What bariatric surgical procedure is recommended for long-term weight loss? A) Gastric banding B) Vertical-banded gastroplasty C) Roux-en-Y D) Biliopancreatic diversion

Roux-en-Y The Roux-en-Y gastric bypass is recommended for long-term weight loss. It is a combined restrictive and malabsorptive procedure. Therefore options A, B, and D are incorrect.

A nurse in the Postanesthesia Care Unit admits a patient following resection of a gastric tumor. The patient should be placed in which position to facilitate both patient comfort and gastric emptying? A) Semi-Fowler's B) Dorsal recumbent C) Side-lying D) Left Simm's

Semi-Fowler's Positioning the patient in a Fowler's position postoperatively promotes comfort and facilitates emptying of the stomach following gastric surgery. Any position that involves lying down delays stomach emptying and is not recommended for this type of patient.

A health care provider counsels a patient about bariatric surgery. He recommends the Roux-en-Y gastric bypass. The nurse explains to the patient that this procedure involves which of the following?

Separation of the jejunum with an anastomosis The Roux-en-Y gastric bypass is recommended for long-term weight loss because it uses a combined restrictive and malabsorptive procedure. Refer to Figure 23-3 (A to D) in the text.

What is the Zollinger-Ellison syndrome? A) Severe peptic ulcers, extreme gastric hyperacidity, and gastrin-secreting benign or malignant tumors of the pancreas B) Gastrin-secreting tumors of the duodenum, extreme gastric hypoacidity, and severe peptic ulcers C) Malignant tumors of the pancreas, severe peptic ulcers, extreme gastric hyperacidity D) Severe peptic ulcers, extreme gastric hypoacidity, and gastrin-secreting benign tumors of the pancreas

Severe peptic ulcers, extreme gastric hyperacidity, and gastrin-secreting benign or malignant tumors of the pancreas Peptic ulcers are found in rare cases in patients with tumors that cause secretion of excessive amounts of the hormone gastrin. The Zollinger-Ellison syndrome (ZES) consists of severe peptic ulcers, extreme gastric hyperacidity, and gastrin-secreting benign or malignant tumors of the pancreas. This makes options B, C and D incorrect.

A patient who experienced a gastric hemorrhage has had the bleeding controlled. The patient's condition is now stable. The nurse caring for this patient should have as her priority frequent assessment of the patient for what? A) Tachycardia, hypotension, and tachypnea B) Tarry, foul-smelling stools C) "Coffee ground" material in nasogastric tube drainage D) Irritation of the nares and nasal mucosa

Tachycardia, hypotension, and tachypnea Tachycardia, hypotension, and tachypnea are signs of recurrent bleeding. Patients who have had one GI bleed are at risk for reoccurrence. Tarry stools and presence of coffee ground material in drainage are expected findings after a hemorrhage. The NG tube may be associated with nasal irritation. While this is an important concern related to patient comfort, it is not a nursing priority

Barium study of the upper GI tract may show; however, endoscopy is the preferred diagnostic procedure because it allows direct visualization. Through endoscopy, a biopsy of the gastric mucosa and of any suspicious lesions can be obtained. Endoscopy may reveal lesions that, because of their size or location, are not evident on x-ray studies.

Ulcers ,inflammatory changes, and lesions

The nurse is inserting a nasoenteric tube for a patient with a paralytic ileus. How long does the nurse anticipate the tube will be required? (Select all that apply.)

Until bowel sound is present Until flatus is passed Until peristalsis is resumed

A nurse is providing care for a client recovering from gastric bypass surgery. During assessment, the client exhibits pallor, perspiration, palpitations, headache, and feelings of warmth, dizziness, and drowsiness. The client reports eating 90 minutes ago. The nurse suspects:

Vasomotor symptoms associated with dumping syndrome

A nurse is providing care for a client recovering from gastric bypass surgery. During assessment, the client exhibits pallor, perspiration, palpitations, headache, and feelings of warmth, dizziness, and drowsiness. The client reports eating 90 minutes ago. What will the nurse suspect? Vasomotor symptoms associated with dumping syndrome Dehiscence of the surgical wound Peritonitis A normal reaction to surgery

Vasomotor symptoms associated with dumping syndrome Early manifestations of dumping syndrome occur 15 to 30 minutes after eating. Signs and symptoms include vertigo, tachycardia, syncope, sweating, pallor, palpitations, diarrhea, nausea, and the desire to lie down. Dehiscence of the surgical wound is characterized by pain and a pulling or popping feeling at the surgical site. Peritonitis presents with a rigid, board-like abdomen, tenderness, and fever. The client's signs and symptoms aren't a normal reaction to surgery.

A nurse is providing care for a client recovering from gastric bypass surgery. During assessment, the client exhibits pallor, perspiration, palpitations, headache, and feelings of warmth, dizziness, and drowsiness. The client reports eating 90 minutes ago. The nurse suspects:

Vasomotor symptoms associated with dumping syndrome Early manifestations of dumping syndrome occur 15 to 30 minutes after eating. Signs and symptoms include vertigo, tachycardia, syncope, sweating, pallor, palpitations, diarrhea, nausea, and the desire to lie down. Dehiscence of the surgical wound is characterized by pain and a pulling or popping feeling at the surgical site. Peritonitis presents with a rigid, boardlike abdomen, tenderness, and fever. The client's signs and symptoms aren't a normal reaction to surgery.

A client who can't tolerate oral feedings begins receiving intermittent enteral feedings. When monitoring for evidence of intolerance to these feedings, the nurse must remain alert for:

diaphoresis, vomiting, and diarrhea.

Which statement correctly identifies a difference between duodenal and gastric ulcers?

Vomiting is uncommon in clients with duodenal ulcers. Vomiting is uncommon in clients diagnosed with duodenal ulcer. Malignancy is associated with a gastric ulcer. Weight gain may occur with a duodenal ulcer. Duodenal ulcers cause hypersecretion of stomach acid.

A patient has just been diagnosed with gastric cancer. The nurse is taking a history on this patient. What would be an important question to ask? A) What is the patient's smoking and alcohol history? B) What is the patient's work history? C) What is the patient's level of fatigue? D) What is the family history of smoking and alcohol use?

What is the patient's smoking and alcohol history? Other health information to obtain includes the patient's smoking and alcohol history and family history (eg, any first- or second-degree relatives with gastric or other cancer). The patient's work history would not be a question asked at this time, and neither would the family history of alcohol and tobacco use. The patient's level of fatigue is not a question asked when taking a history

After a client received a diagnosis of gastric cancer, the surgical team decides that a Billroth II would be the best approach to treatment. The nurse explains to the family that this procedure involves:

Wide resection of the middle and distal portions of the stomach with removal of about 75% of the stomach The Billroth I involves a limited resection and offers a lower cure rate than the Billroth II. The Billroth II procedure is a wider resection that involves removing approximately 75% of the stomach and decreases the possibility of lymph node spread or metastatic recurrence. A proximal subtotal gastrectomy may be performed for a resectable tumor located in the proximal portion of the stomach or cardia. A total gastrectomy or an esophagogastrectomy is usually performed in place of this procedure to achieve a more extensive resection.

hypersecretion of acid may be controlled with high doses of H2 receptor antagonists. These clients may require twice the normal dose, and dosages usually need to be increased with prolonged use. Octreotide (Sandostatin), a medication that suppresses gastrin levels, also may be prescribed.

Zollinger-Ellison syndrome (ZES)

A client is admitted to the health care facility with a diagnosis of a bleeding gastric ulcer. The nurse expects this client's stools to be:

black and tarry. Black, tarry stools are a sign of bleeding high in the GI tract, as from a gastric ulcer, and result from the action of digestive enzymes on the blood. Vomitus associated with upper GI tract bleeding commonly is described as coffee-ground-like. Clay-colored stools are associated with biliary obstruction. Bright red stools indicate lower GI tract bleeding.

The nurse is creating a discharge plan of care for a client with a peptic ulcer. The nurse tells the client to avoid

decaffeinated coffee. The nurse should include avoidance of decaffeinated coffee in the client's discharge teaching plan. Decaffeinated coffee is avoided to keep from overstimulating acid secretion.

A nurse is providing follow-up teaching at a clinic visit for a client recovering from gastric resection. The client reports sweating, diarrhea, nausea, palpitations, and the desire to lie down 15 to 30 minutes after meals. The nurse suspects the client has:

dumping syndrome.

A nurse is providing follow-up teaching at a clinic visit for a client recovering from gastric resection. The client reports sweating, diarrhea, nausea, palpitations, and the desire to lie down 15 to 30 minutes after meals. The nurse suspects the client has

dumping syndrome. Early manifestations of dumping syndrome occur 15 to 30 minutes after eating. Signs and symptoms include vertigo, tachycardia, syncope, sweating, pallor, palpitations, diarrhea, nausea, and the desire to lie down. Dehiscence of the surgical wound is characterized by pain and a pulling or popping feeling at the surgical site. Peritonitis presents with a rigid, boardlike abdomen, tenderness, and fever. The client's signs and symptoms aren't a normal reaction to surgery.

The Zollinger-Ellison syndrome (ZES) consists of severe peptic ulcers, extreme gastric hyperacidity, and gastrin-secreting benign or malignant tumors of the pancreas. The nurse recognizes that an agent that is used to decrease bleeding and decrease gastric acid secretions is: octreotide (Sandostatin) vasopressin (Pitressin) omeprazole (Prilosec) ranitidine (Zantac)

octreotide (Sandostatin) For patients with ZES, hypersecretion of acid may be controlled with high doses of H2 receptor antagonists. These clients may require twice the normal dose, and dosages usually need to be increased with prolonged use. Octreotide (Sandostatin), a medication that suppresses gastrin levels, also may be prescribed.

A nurse is caring for a client who is undergoing a diagnostic workup for a suspected GI problem. The client reports gnawing epigastric pain following meals and heartburn. The nurse suspects the client has:

peptic ulcer disease. Peptic ulcer disease is characterized by dull, gnawing pain in the midepigastrium or the back that worsens with eating. Ulcerative colitis is characterized by exacerbations and remissions of severe bloody diarrhea. Appendicitis is characterized by epigastric or umbilical pain along with nausea, vomiting, and low-grade fever. Pain caused by diverticulitis is in the left lower quadrant and has a moderate onset. It's accompanied by nausea, vomiting, fever, and chills.

Between 40 and 70 years, but can occur in younger people. Men have a higher incidence than women. Native Americans, Hispanic Americans, and African Americans are twice as likely as Caucasian Americans to develop . A diet high in smoked, salted, or pickled foods and low in fruits and vegetables may increase the risk . Other factors related to the incidence include chronic inflammation of the stomach, H. pylori infection, pernicious anemia, smoking, achlorhydria, gastric ulcers, subtotal gastrectomy more than 20 years ago, and genetics.

typical client with gastric cancer

To prevent gastroesophageal reflux in a client with hiatal hernia, the nurse should provide which discharge instruction?

"Avoid coffee and alcoholic beverages."

A nurse is performing discharge teaching with a client who had a total gastrectomy. Which statement indicates the need for further teaching? A. "I will call my physician if I begin to have abdominal pain." B. "I will have to take vitamin B12 shots up to 1 year after surgery." C. "I will weight myself each day and record the weight." D. "I'm going to visit my pastor weekly for a while."

"I will have to take vitamin B12 shots up to 1 year after surgery." After a total gastrectomy, a client will need to take vitamin B12 shots for life. Dietary B12 is absorbed in the stomach, and the inability to absorb it could lead to pernicious anemia

A client has recently been diagnosed with gastric cancer. He has a history of tobacco use and was diagnosed 10 years ago with pernicious anemia. He and his family are shocked about the possibility of this diagnosis because he has been asymptomatic prior to recent complaints of pain and multiple gastrointestinal symptoms. On palpation, the nurse notes two signs that confirm metastasis to the liver. Which of the following are signs? Choose all that apply.

-Ascites -Hepatomegaly The physical examination is usually not helpful in detecting the cancer because most early gastric tumors are not palpable. Advanced gastric cancer may be palpable as a mass. Ascites and hepatomegaly (enlarged liver) may be apparent if the cancer cells have metastasized to the liver. Palpable nodules around the umbilicus, called Sister Mary Joseph's nodules, are a sign of a GI malignancy, usually a gastric cancer. A distended bladder is not significant. Petechiae at the palpation site is a distractor for the question.

Perforation is the erosion of the ulcer through the gastric serosa into the peritoneal cavity without warning. It is an abdominal catastrophe and requires immediate surgery. Penetration is erosion of the ulcer through the gastric serosa into adjacent structures such as the pancreas, biliary tract, or gastrohepatic omentum. What are the signs and symptoms of perforation? (Mark all that apply.) A) Collapse B) Hypertension C) Pain referred to the right shoulder D) Vomiting E) Bradycardia

-Collapse -Pain referred to the right shoulder -Vomiting Signs and symptoms of perforation include the following: sudden, severe upper abdominal pain (persisting and increasing in intensity); pain may be referred to the shoulders, especially the right shoulder, because of irritation of the phrenic nerve in the diaphragm; vomiting; collapse (fainting); extremely tender and rigid (boardlike) abdomen; and hypotension and tachycardia, indicating shock.

A client with active schizophrenia has developed acute gastritis after ingesting a strongly alkaline solution during a psychotic episode. Corrosion is extensive. Which of the following emergency treatments might the team working with this client use? Select all that apply.

-Diluted lemon juice -Diluted vinegar Emergency treatment consists of diluting and neutralizing the offending agent. To neutralize acids, common antacids (eg, aluminum hydroxide) are used; to neutralize an alkali, diluted lemon juice or diluted vinegar is used. If corrosion is extensive or severe, emetics such as syrup of ipecac and lavage are avoided because of the danger of perforation and damage to the esophagus.

The nurse reviews dietary guidelines with a patient who had a gastric banding. Which of the following teaching points are included? Select all that apply.

-Do not eat and drink at the same time. -Drink plenty of water, from 90 minutes after each meal to 15 minutes before each meal. -Avoid fruit drinks and soda. Total meal size should be restricted to less than 8 oz or 240 mL. Three meals a day are recommended. Refer to Box 23-2 in the text.

A client returns from the operating room after receiving extensive abdominal surgery. He has 1,000 mL of lactated Ringer's solution infusing via a central line. The physician orders the IV fluid to be infused at 125 mL/hr plus the total output of the previous hour. The drip factor of the tubing is 15 gtt/min, and the output for the previous hour was 75 mL via Foley catheter, 50 mL via nasogastric tube, and 10 mL via Jackson Pratt tube. For how many drops per mintue should the nurse set the IV flow rate to deliver the correct amount of fluid? Enter the correct number ONLY.

65 First, calculate the volume to be infused (in milliliters): 75 mL + 50 mL + 10 mL = 135 mL total output for the previous hour; 135 mL + 125 mL ordered as a constant flow = 260 mL to be infused over the next hour Next use the formula Volume to be infused/Total minutes to be infused x Drip factor = Drops per min In this case, 260 mL divided by 60 min x 15 gtt/min = 65 gtt/min

When caring for a client with an acute exacerbation of a peptic ulcer, the nurse finds the client doubled up in bed with severe pain in the right shoulder. What is the initial appropriate action by the nurse? Notify the health care provider. Irrigate the client's NG tube. Place the client in the high-Fowler's position. Assess the client's abdomen and vital signs.

Assess the client's abdomen and vital signs. Signs and symptoms of perforation includes sudden, severe upper abdominal pain (persisting and increasing in intensity); pain may be referred to the shoulders, especially the right shoulder, because of irritation of the phrenic nerve in the diaphragm. The nurse should assess the vital signs and abdomen prior to notifying the physician. Irrigation of the NG tube should not be performed because the additional fluid may be spilled into the peritoneal cavity, and the client should be placed in a position of comfort, usually on the side with the head slightly elevated.

The nurse is doing triage at the community clinic when a middle-aged patient presents with abdominal pains and heartburn. The patient states the symptoms have persisted for several days following a particularly spicy meal. When assessing the patient, the nurse notes the patient has a history of acute gastritis. What complication would the nurse be particularly likely to assess for? A) Esophageal or pyloric obstruction related to scarring B) Acute systemic infection related to peritonitis C) Gastric hyperacidity related to excessive gastrin secretion D) Chronic referred pain in the right shoulder

Esophageal or pyloric obstruction related to scarring Acute gastritis can result from dietary indiscretion, and may lead to scarring and stenosis often requiring dilation. Acute systemic infection occurs following perforation. Perforation is not a common occurrence following ingestion of a corrosive substance. Gastrin is a hormone secreted in the stomach. It is not normally found in the esophagus. Chronic referred pain to the right shoulder is a symptom of peptic ulcer disease, but would not be an expected finding for a patient who has ingested a corrosive substance.

A client is recovering from gastric surgery. Which of the following is the correct position for the nurse to place this client?

Fowler's Placing the client in the Fowler's position after gastric surgery promotes comfort and allows emptying of the stomach.

A client with severe peptic ulcer disease has undergone surgery and is several hours postoperative. During assessment, the nurse notes that the client has developed cool skin, tachycardia, labored breathing, and appears to be confused. Which complication has the client most likely developed? Hemorrhage Penetration Perforation Pyloric obstruction

Hemorrhage Signs of hemorrhage following surgery include cool skin, confusion, increased heart rate, labored breathing, and blood in the stool. Signs of penetration and perforation are severe abdominal pain, rigid and tender abdomen, vomiting, elevated temperature, and increased heart rate. Indicators of pyloric obstruction are nausea, vomiting, distended abdomen, and abdominal pain.

Which of the following is considered an early symptom of gastric cancer? A. Pain relieved by antacids B. Weight loss C. Dyspepsia D. Bloating after meals

Pain relieved by antacids Symptoms of early disease, such as pain relieved by antacids, resemble those of benign ulcers and are seldom definitive. Symptoms of progressive disease include weight loss, bloating after meals, and dyspepsia.

A client is in the hospital for the treatment of peptic ulcer disease. The client reports vomiting and a sudden severe pain in the abdomen. The nurse then assesses a board-like abdomen. What does the nurse suspect these symptoms indicate? Ineffective treatment for the peptic ulcer A reaction to the medication given for the ulcer Gastric penetration Perforation of the peptic ulcer

Perforation of the peptic ulcer Signs and symptoms of perforation include the following: Sudden, severe upper abdominal pain (persisting and increasing in intensity), which may be referred to the shoulders, especially the right shoulder, because of irritation of the phrenic nerve in the diaphragm; vomiting; collapse (fainting); extremely tender and rigid (boardlike) abdomen; and hypotension and tachycardia, indicating shock.

sudden, severe upper abdominal pain (persisting and increasing in intensity); pain may be referred to the shoulders, especially the right shoulder, because of irritation of the phrenic nerve in the diaphragm. The nurse should assess the vital signs and abdomen prior to notifying the physician. Irrigation of the NG tube should not be performed because the additional fluid may be spilled into the peritoneal cavity, and the client should be placed in a position of comfort, usually on the side with the head slightly elevated.

Signs and symptoms of perforation includes

A client is recovering from gastric surgery. Toward what goal should the nurse progress the client's enteral intake?

Six small meals daily with 120 mL fluid between meals After the return of bowel sounds and removal of the nasogastric tube, the nurse may give fluids, followed by food in small portions. Foods are gradually added until the client can eat six small meals a day and drink 120 mL of fluid between meals.

A client is recovering from gastric surgery. Toward what goal should the nurse progress the client's enteral intake? Three meals and 120 ml fluid daily Three meals and three snacks and 120 mL fluid daily Six small meals and 120 mL fluid daily Six small meals daily with 120 mL fluid between meals

Six small meals daily with 120 mL fluid between meals After the return of bowel sounds and removal of the nasogastric tube, the nurse may give fluids, followed by food in small portions. Foods are gradually added until the client can eat six small meals a day and drink 120 mL of fluid between meals.term-27

A client is preparing for discharge to home following a partial gastrectomy and vagotomy. Which is the best rationale for the client being taught to lie down for 30 minutes after each meal?

Slows gastric emptying

A client with peptic ulcer disease wants to know nonpharmacologic ways to prevent recurrence. Which of the following measures would the nurse recommend? Select all that apply. Substitution of coffee with decaffeinated products Smoking cessation Eating whenever hungry Avoidance of alcohol Following a regular schedule for rest, relaxation, and meals

Smoking cessation Avoidance of alcohol Following a regular schedule for rest, relaxation, and meals The likelihood of recurrence is reduced if the client avoids smoking, coffee (including decaffeinated coffee) and other caffeinated beverages, and alcohol. It is important to counsel the client to eat meals at regular times and in a relaxed setting and to avoid overeating.

The nurse is caring for a client with chronic gastritis. The nurse monitors the client knowing that this client is at risk for which vitamin deficiency?

Vitamin B12 Clients with chronic gastritis from vitamin deficiency usually have evidence of malabsorption of vitamin B12 caused by the production of antibodies that interfere with the binding of vitamin B12 to intrinsic factor. However, some clients with chronic gastritis have no symptoms. Vitamins A, C, and E are not affected by gastritis.

A nurse is monitoring a client with peptic ulcer disease. Which of the following assessment findings would most likely indicate perforation of the ulcer? Choose all that apply.

-Tachycardia -Hypotension -A rigid, board-like abdomen Signs and symptoms of perforation include sudden, severe upper abdominal pain (persisting and increasing in intensity); pain, which may be referred to the shoulders, especially the right shoulder, because of irritation of the phrenic nerve in the diaphragm; vomiting; collapse (fainting); extremely tender and rigid (board-like) abdomen; and hypotension and tachycardia, indicating shock. Perforation is a surgical emergency.

A patient asks the Home Health Nurse what the distressing symptoms of dumping syndrome result from? What physiologic occurrence would the nurse explain? A) Irritation of the phrenic nerve due to diaphragmatic pressure B) Chronic malabsorption of iron and vitamins A and C C) Reflux of bile into the distal esophagus D) Osmotic transport of extracellular fluid into the gastrointestinal tract

Osmotic transport of extracellular fluid into the gastrointestinal tract Following gastric surgery, the gastric remnant is anastamosed to the jejunum. When substances high in carbohydrates and electrolytes are ingested rapidly, they pass directly into the jejunum. Extracellular fluid from the bloodstream is drawn into the jejunum to dilute these hypertonic intestinal contents. Irritation of the phrenic nerve causes hiccups. Reflux of bile is an etiologic factor associated with the development of GERD. Patients who have undergone partial gastrectomy or bariatric surgery may absorb vitamins and minerals less effectively; however, this change is unrelated to the occurrence of dumping syndrome.

The healthcare provider of a client with oral cancer has ordered the placement of a GI tube to provide nutrition and to deliver medications. What would be the preferred route?

nasogastric intubation


Conjuntos de estudio relacionados

Test 1 PSY 3450 (Cross-Cultural Research)

View Set

"The Price We Pay"? Pornography and Harm (Brison)

View Set

Benchmark Fraction, Decimal, Percents

View Set

ACT practice math portion 2023-2024

View Set

Chapter 2 - Network Infrastructure and Documentation

View Set